2023 COMMUNITY HEALTH RN V3 (VERSION 3) EXIT HESI GUARANTEED A+ TEST BANK|AGRADE

Enalapril maleate (Vasotec) is prescribed for a hospitalized client. Which assessment does the nurse perform as a priority before administering the medication?
A) Checking the client’s blood pressure
B) checking the client’s peripheral pulses
C) checking the most recent potassium level
D) checking the client’s intake and output record for the last 24 hours
A) Checking the client’s blood pressure

Rationale–enalapril maleate is an angiotensin-converting enzyme (ACE) inhibitor used to treat hypertension. One common side effect is postural hypotension. Therefore the nurse would check the client’s blood pressure immediately before administering each dose.

A client is scheduled to undergo an upper gastrointestinal (GI) series, and the nurse provides instructions to the client about the test. Which statement by the client indicates a need for further instruction?

A) “The test will take about 30 minutes”
B) “I need to fast for 8 hours before the test”
C) “I need to drink citrate of magnesia the night before the test and give myself a Fleet enema on the morning of the test.”
D) “I need to take a laxative after the test is completed, because the liquid that I’ll have to drink for the test can be constipating.”
C) “I need to drink citrate of magnesia the night before the test and give myself a Fleet enema on the morning of the test.”

Rationale
An upper GI series involves visualization of the esophagus, duodenum, and upper
jejunum by means of the use of a contrast medium. It involves swallowing a contrast
medium (usually barium), which is administered in a flavored milkshake. Films are taken
at intervals during the test, which takes about 30 minutes. No special preparation is
necessary before a GI series, except that NPO status must be maintained for 8 hours
before the test. After an upper GI series, the client is prescribed a laxative to hasten
elimination of the barium. Barium that remains in the colon may become hard and
difficult to expel, leading to fecal impaction.

A nurse on the evening shift checks a physician’s prescriptions and notes that the dose of a prescribed medication is higher than the normal dose. The nurse calls the physician’s answering service and is told that the physician if off for the night and will be available the next morning. The nurse should:
A) call the nursing supervisor
B) Ask the answering service to contact the on-call physician
C) Withhold the medication until the physician can be reached in the morning
D) Administer the medication but consult the physician when he becomes available
B) Ask the answering service to contact the on-call physician

An emergency department (ED) nurse is monitoring a client with suspected acute
myocardial infarction (MI) who is awaiting transfer to the coronary intensive care unit.
The nurse notes the sudden onset of premature ventricular contractions (PVCs) on the
monitor, checks the client’s carotid pulse, and determines that the PVCs are not
resulting in perfusion. The appropriate action by the nurse is:
A. Documenting the findings
B. Asking the ED physician to check the client
C. Continuing to monitor the client’s cardiac status
D. Informing the client that PVCs are expected after an MI
B. Asking the ED physician to check the client

NPO status is imposed 8 hours before the procedure on a client scheduled to undergo
electroconvulsive therapy (ECT) at 1 p.m. On the morning of the procedure, the nurse
checks the client’s record and notes that the client routinely takes an oral
antihypertensive medication each morning. The nurse should:
A. Administer the antihypertensive with a small sip of water
B. Withhold the antihypertensive and administer it at bedtime
C. Administer the medication by way of the intravenous (IV) route
D. Hold the antihypertensive and resume its administration on the day after the ECT
Administer the antihypertensive with a small sip of water

In planning care for a 6 month-old infant, what must the nurse provide to assist in the development of trust?
A) Food
B) Warmth
C) Security
D) Comfort
C) Security

A nurse has just received a medication order which is not legible. Which statement best reflects assertive communication?
A) “I cannot give this medication as it is written. I have no idea of what you mean.”
B) “Would you please clarify what you have written so I am sure I am reading it
correctly?”
C) “I am having difficulty reading your handwriting. It would save me time if you would be more careful.”
D) “Please print in the future so I do not have to spend extra time attempting to read your writing.”
B) “Would you please clarify what you have written so I am sure I am reading it
correctly?”

What is the most important consideration when teaching parents how to reduce risks in the home?
A) Age and knowledge level of the parents
B) Proximity to emergency services
C) Number of children in the home
D) Age of children in the home
D) Age of children in the home

A 35 year-old client with sickle cell crisis is talking on the telephone but stops as the nurse enters the room to request something for pain. The nurse should
A) Administer a placebo
B) Encourage increased fluid intake
C) Administer the prescribed analgesia
D) Recommend relaxation exercises for pain control
C) Administer the prescribed analgesia

While caring for a toddler with croup, which initial sign of croup requires the nurse’s immediate attention?
A) Respiratory rate of 42
B) Lethargy for the past hour
C) Apical pulse of 54
D) Coughing up copious secretions
A) Respiratory rate of 42

A client is admitted with low T3 and T4 levels and an elevated TSH level. On initial assessment, the nurse would anticipate which of the following assessment findings?
A) Lethargy
B) Heat intolerance
C) Diarrhea
D) Skin eruptions
A) Lethargy

The emergency room nurse admits a child who experienced a seizure at school. The father comments that this is the first occurrence, and denies any family history of epilepsy. What is the best response by the nurse?
A) “Do not worry. Epilepsy can be treated with medications.”
B) “The seizure may or may not mean your child has epilepsy.”
C) “Since this was the first convulsion, it may not happen again.”
D) “Long term treatment will prevent future seizures.”
B) “The seizure may or may not mean your child has epilepsy.”

Alcohol and drug abuse impairs judgment and increases risk taking behavior. What nursing diagnosis best applies?
A) Risk for injury
B) Risk for knowledge deficit
C) Altered thought process
D) Disturbance in self-esteem
A) Risk for injury

Which these findings would the nurse more closely associate with anemia in a 10 month-old infant?
A) Hemoglobin level of 12 g/dI
B) Pale mucosa of the eyelids and lips
C) Hypoactivity
D) A heart rate between 140 to 160
B) Pale mucosa of the eyelids and lips

The nurse is caring for a client in hypertensive crisis in an intensive care unit. The priority assessment in the first hour of care is
A) Heart rate
B) Pedal pulses
C) Lung sounds
D) Pupil responses
D) Pupil responses

Which of these clients who are all in the terminal stage of cancer is least appropriate to suggest the use of patient controlled analgesia (PCA) with a pump?
A) A young adult with a history of Down’s syndrome
B) A teenager who reads at a 4th grade level
C) An elderly client with numerous arthritic nodules on the hands
D) A preschooler with intermittent episodes of alertness
D) A preschooler with intermittent episodes of alertness

The nurse is about to assess a 6 month-old child with nonorganic failure-to thrive (NOFTT). Upon entering the room, the nurse would expect the baby to be
A) Irritable and “colicky” with no attempts to pull to standing
B) Alert, laughing and playing with a rattle, sitting with support
C) Skin color dusky with poor skin turgor over abdomen
D) Pale, thin arms and legs, uninterested in surroundings
D) Pale, thin arms and legs, uninterested in surroundings

As the nurse is speaking with a group of teens which of these side effects of chemotherapy for cancer would the nurse expect this group to be more interested in during the discussion?
A) Mouth sores
B) Fatigue
C) Diarrhea
D) Hair loss
D) Hair loss

While caring for a client who was admitted with myocardial infarction (MI) 2 days ago, the nurse notes today’s temperature is 101.1 degrees Fahrenheit (38.5 degreesCelsius). The appropriate nursing intervention is to
A) Call the health care provider immediately
B) Administer acetaminophen as ordered as this is normal at this time
C) Send blood, urine and sputum for culture
D) Increase the client’s fluid intake
B) Administer acetaminophen as ordered as this is normal at this time

A client is admitted for first and second degree burns on the face, neck, anterior chest and hands. The nurse’s priority should be
A) Cover the areas with dry sterile dressings
B) Assess for dyspnea or stridor
C) Initiate intravenous therapy
D) Administer pain medication
B) Assess for dyspnea or stridor

Which of these clients who call the community health clinic would the nurse ask to come in that day to be seen by the health care provider?
A) I started my period and now my urine has turned bright red.
B) I am an diabetic and today I have been going to the bathroom every hour.
C) I was started on medicine yesterday for a urine infection. Now my lower belly hurts when I go to the bathroom.
D) I went to the bathroom and my urine looked very red and it didn’t hurt when I went.
D) I went to the bathroom and my urine looked very red and it didn’t hurt when I went.

Which of these parents’ comment for a newborn would most likely reveal an initial finding of a suspected pyloric stenosis?
A) I noticed a little lump a little above the belly button.
B) The baby seems hungry all the time.
C) Mild vomiting that progressed to vomiting shooting across the room.
D) Irritation and spitting up immediately after feedings.
C) Mild vomiting that progressed to vomiting shooting across the room.

The nurse is assessing a child for clinical manifestations of iron deficiency anemia.
Which factor would the nurse recognize as cause for the findings?
A) Decreased cardiac output
B) Tissue hypoxia
C) Cerebral edema
D) Reduced oxygen saturation
B) Tissue hypoxia

The nurse would expect the cystic fibrosis client to receive supplemental pancreatic
enzymes along with a diet
A) High in carbohydrates and proteins
B) Low in carbohydrates and proteins
C) High in carbohydrates, low in proteins
D) Low in carbohydrates, high in proteins
A) High in carbohydrates and proteins

In evaluating the growth of a 12 month-old child, which of these findings would the
nurse expect to be present in the infant?
A) Increased 10% in height
B) 2 deciduous teeth
C) Tripled the birth weight
D) Head > chest circumference
C) Tripled the birth weight

A Hispanic client in the postpartum period refuses the hospital food because it is
“cold.” The best initial action by the nurse is to
A) 1Have the unlicensed assistive personnel (UAP) reheat the food if the client wishes
B) Ask the client what foods are acceptable or bad
C) Encourage her to eat for healing and strength
D) Schedule the dietitian to meet with the client as soon as possible
B) Ask the client what foods are acceptable or bad

The father of an 8 month-old infant asks the nurse if his infant’s vocalizations are
normal for his age. Which of the following would the nurse expect at this age?
A) Cooing
B) Imitation of sounds
C) Throaty sounds
D) Laughter
B) Imitation of sounds

The nurse should recognize that physical dependence is accompanied by what
findings when alcohol consumption is first reduced or ended?
A) Seizures
B) Withdrawal
C) Craving
D) Marked tolerance
B) Withdrawal

Immediately following an acute battering incident in a violent relationship, the
batterer may respond to the partner’s injuries by
A) Seeking medical help for the victim’s injuries
B) Minimizing the episode and underestimating the victim’s injuries
C) Contacting a close friend and asking for help
D) Being very remorseful and assisting the victim with medical care
B) Minimizing the episode and underestimating the victim’s injuries

A client with pneumococcal pneumonia had been started on antibiotics 16 hours ago.During the nurse’s initial evening rounds the nurse notices a foul smell in the room. The client makes all of these statements during their conversation. Which statement would alert the nurse to a complication?
A) “I have a sharp pain in my chest when I take a breath.
“B) “I have been coughing up foul-tasting, brown, thick sputum.
” C) “I have been sweating all day.
“D) “I feel hot off and on.”
“B) “I have been coughing up foul-tasting, brown, thick sputum.

The nurse is performing an assessment on a client in congestive heart failure. Auscultation of the heart is most likely to reveal
A) S3 ventricular gallop
B) Apical click
C) Systolic murmur
D) Split S2
A) S3 ventricular gallop

Which of these observations made by the nurse during an excretory urogram indicate a complicaton?
A) The client complains of a salty taste in the mouth when the dye is injected
B) The client’s entire body turns a bright red color
C) The client states “I have a feeling of getting warm.”
D) The client gags and complains ” I am getting sick.”
B) The client’s entire body turns a bright red color

A client is diagnosed with a spontaneous pneumothorax necessitating the insertion of a chest tube. What is the best explanation for the nurse to provide this client?
A) “The tube will drain fluid from your chest.
“B) “The tube will remove excess air from your chest.”
C) “The tube controls the amount of air that enters your chest.
” D) “The tube will seal the hole in your lung.”
“B) “The tube will remove excess air from your chest.”

The nurse is reviewing laboratory results on a client with acute renal failure. Which one of the following should be reported immediately?
A) Blood urea nitrogen 50 mg/dl
B) Hemoglobin of 10.3 mg/dl
C) Venous blood pH 7.30
D) Serum potassium 6 mEq/L
D) Serum potassium 6 mEq/L

The nurse is caring for a client undergoing the placement of a central venous catheter line. Which of the following would require the nurse’s immediate attention?
A) Pallor
B) Increased temperature
C) Dyspnea
D) Involuntary muscle spasms
C) Dyspnea

The nurse is performing a physical assessment on a client who just had an endotracheal tube inserted. Which finding would call for immediate action by the nurse?
A) Breath sounds can be heard bilaterally
B) Mist is visible in the T-Piece
C) Pulse oximetry of 88
D) Client is unable to speak
C) Pulse oximetry of 88

A nurse checks a client who is on a volume-cycled ventilator. Which finding indicates that the client may need suctioning?A) Drowsiness
B) Complaint of nausea
C) Pulse rate of 92
D) Restlessness
D) Restlessness

During the evaluation phase for a client, the nurse should focus on
A) All finding of physical and psychosocial stressors of the client and in the family
B) The client’s status, progress toward goal achievement, and ongoing re-evaluation
C) Setting short and long-term goals to insure continuity of care from hospital to home
D) Select interventions that are measurable and achievable within selected timeframes
B) The client’s status, progress toward goal achievement, and ongoing re-evaluation

The school nurse suspects that a third grade child might have Attention Deficit Hyperactivity Disorder. Prior to referring the child for further evaluation, the nurse should
A) Observe the child’s behavior on at least 2 occasions
B) Consult with the teacher about how to control impulsivity
C) Compile a history of behavior patterns and developmental accomplishments
D) Compare the child’s behavior with classic signs and symptoms
C) Compile a history of behavior patterns and developmental accomplishments

Which of the actions suggested to the RN by the PN during a planning conference for a 10 month-old infant admitted 2 hours ago with bacterial meningitis would be acceptable to add to the plan of care?
A) Measure head circumference
B) Place in airborne isolation
C) Provide passive range of motion
D) Provide an over-the-crib protective top
A) Measure head circumference

A client is admitted with a diagnosis of hepatitis B. In reviewing the initial laboratory
results, the nurse would expect to find elevation in which of the following values?
A) Blood urea nitrogen
B) Acid phosphatase
C) Bilirubin
D) Sedimentation Rate
C) Bilirubin

The nurse is discussing nutritional requirements with the parents of an 18 month-old
child. Which of these statements about milk consumption is correct?
A) May drink as much milk as desired
B) Can have milk mixed with other foods
C) Will benefit from fat-free cow’s milk
D) Should be limited to 3-4 cups of milk daily
D) Should be limited to 3-4 cups of milk daily

The nurse is talking with a client. The client abruptly says to the nurse, “The moon is full. Astronauts walk on the moon. Walking is a good health habit.” The client’s behavior most likely indicates
A) Neologisms
B) Dissociation
C) Flight of ideas
D) Word salad
C) Flight of ideas

A mother asks about expected motor skills for a 3 year-old child. Which of the
following would the nurse emphasize as normal at this age? A) Jumping rope
B) Tying shoelaces
C) Riding a tricycle
D) Playing hopscotch
C) Riding a tricycle

A home health nurse is caring for a client with a pressure sore that is red, with serous drainage, is 2 inches in diameter with loss of subcutaneous tissue. The appropriate dressing for this wound is
A) A transparent film dressing
B) Wet dressing with debridement granules
C) Wet to dry with hydrogen peroxide
D) Moist saline dressing
D) Moist saline dressing

The nurse enters a 2 year-old child’s hospital room in order to administer an oral medication. When the child is asked if he is ready to take his medicine, he immediately says, “No!” What would be the most appropriate next action?
A) Leave the room and return five minutes later and give the medicine
B) Explain to the child that the medicine must be taken now C) Give the medication to the father and ask him to give it
D) Mix the medication with ice cream or applesauce
A) Leave the room and return five minutes later and give the medicine

A nurse is doing pre conceptual counseling with a woman who is planning a pregnancy. Which of the following statements suggests that the client understands the connection between alcohol consumption and fetal alcohol syndrome?
A) “I understand that a glass of wine with dinner is healthy.
“B) “Beer is not really hard alcohol, so I guess I can drink some.
“C) “If I drink, my baby may be harmed before I know I am pregnant.
” D) “Drinking with meals reduces the effects of alcohol.”
“C) “If I drink, my baby may be harmed before I know I am pregnant.

A client has returned from a cardiac catheterization. Which one of the following assessments would indicate the client is experiencing a complication from the procedure?
A) Increased blood pressure
B) Increased heart rate
C) Loss of pulse in the extremity
D) Decreased urine output
C) Loss of pulse in the extremity

A 60 year-old male client had a hernia repair in an outpatient surgery clinic. He is awake and alert, but has not been able to void since he returned from surgery 6 hours ago.He received 1000 mL of IV fluid. Which action would be most likely to help him void?
A) Have him drink several glasses of water
B) Crede’ the bladder from the bottom to the top
C) Assist him to stand by the side of the bed to void
D) Wait 2 hours and have him try to void again
C) Assist him to stand by the side of the bed to void

The nurse is caring for a client who requires a mechanical ventilator for breathing.The high pressure alarm goes off on the ventilator. What is the first action the nurse should perform?
A) Disconnect the client from the ventilator and use a manual resuscitation bag
B) Perform a quick assessment of the client’s condition
C) Call the respiratory therapist for help
D) Press the alarm re-set button on the ventilator
B) Perform a quick assessment of the client’s condition

The nurse is preparing a client who will undergo a myelogram. Which of the following statements by the client indicates a contraindication for this test?
A) “I can’t lie in 1 position for more than thirty minutes.
“B) “I am allergic to shrimp.”
C) “I suffer from claustrophobia.
“D) “I developed a severe headache after a spinal tap.”
“B) “I am allergic to shrimp.”

The health care provider order reads “aspirate nasogastric feeding (NG) tuber every 4 hours and check pH of aspirate.” The pH of the aspirate is 10. Which action should the nurse take?
A) Hold the tube feeding and notify the provider
B) Administer the tube feeding as scheduled
C) Irrigate the tube with diet cola soda
D) Apply intermittent suction to the feeding tube
A) Hold the tube feeding and notify the provider

To prevent unnecessary hypoxia during suctioning of a tracheostomy, the nurse must
A) Apply suction for no more than 10 seconds
B) Maintain sterile technique
C) Lubricate 3 to 4 inches of the catheter tip
D) Withdraw catheter in a circular motion
A) Apply suction for no more than 10 seconds

An antibiotic IM injection for a 2 year-old child is ordered. The total volume of the injection equals 2.0 ml The correct action is to
A) Administer the medication in 2 separate injections
B) Give the medication in the dorsal gluteal site
C) Call to get a smaller volume ordered
D) Check with pharmacy for a liquid form of the medication skip
A) Administer the medication in 2 separate injections

The nurse receives an order to give a client iron by deep injection. The nurse know that the reason for this route is to
A) Enhance absorption of the medication
B) Ensure that the entire dose of medication is given
C) Provide more even distribution of the drug
D) Prevent the drug from tissue irritation Skip
D) Prevent the drug from tissue irritation Skip

A client with heart failure has Lanoxin (digoxin) ordered. What would the nurse expect to find when evaluating for the therapeutic effectiveness of this drug?
A) Diaphoresis with decreased urinary output
B) Increased heart rate with increase respirations
C) Improved respiratory status and increased urinary output D) Decreased chest pain and decreased blood pressure
C) Improved respiratory status and increased urinary output

While providing home care to a client with congestive heart failure, the nurse is asked how long diuretics must be taken. What is the nurse’s best response?
A) “As you urinate more, you will need less medication to control fluid.
“B) “You will have to take this medication for about a year.”
C) “The medication must be continued so the fluid problem is controlled.
“D) “Please talk to your health care provider about medications and treatments.”
C) “The medication must be continued so the fluid problem is controlled.

A client is being discharged with a prescription for chlorpromazine (Thorazine).Before leaving for home, which of these findings should the nurse teach the client to report? A) Change in libido, breast enlargement
B) Sore throat, fever
C) Abdominal pain, nausea, diarrhea
D) Dsypnea, nasal congestion
B) Sore throat, fever

A mother brings her 6-year-old child, who has just stepped on a rusty nail, to the pediatrician’s office. Upon inspection, the nurse notes that the nail went through the shoe and pierced the bottom of the child’s foot. Which action should the nurse implement first?
A. Cleanse the foot with soap and water and apply an antibiotic ointment
B. Provide teaching about the need for a tetanus booster within the next 72 hours.
C. have the mother check the child’s temperature q4h for the next 24 hours
D. transfer the child to the emergency department to receive a gamma globulin
injection
A. Cleanse the foot with soap and water and apply an antibiotic ointment

A 26-year-old female client is admitted to the hospital for treatment of a simple goiter, and levothyroxine sodium (Synthroid) is prescribed. Which symptoms indicate to the nurse that the prescribed dosage is too high for this client? The client experiences:
A. Bradycardia and constipation
B. Lethargy and lack of appetite
C. Muscle cramping and dry, flushed skin
D. Palpitations and shortness of breath
D. Palpitations and shortness of breath

A client with a history of heart failure presents to the clinic with a nausea, vomiting, yellow vision and palpitations. Which finding is most important for the nurse to assess to the client?
Obtain a list of medications taken for cardiac history

The pathophysiological mechanism are responsible for ascites related to liver failure? (Select all that apply)
A. Fluid shifts from intravascular to interstitial area due to decreased serum
protein
B. Increased hydrostatic pressure in portal circulation increases
fluid shifts into abdomen
C. Increased circulating aldosterone levels that increase sodium and water
retention

The nurse is auscultating a client’s heart sounds. Which description should the nurse use to document this sound? (Please listen to the audio first to select the option that applies)
Murmur

A client is admitted with a pressure ulcer in the sacral area. The partial thickness wound is 4cm by 7cm, the wound base is red and moist with no exudate and the surrounding skin is intact. Which of the following coverings is most appropriate for this wound?
A) Transparent dressing
B) Dry sterile dressing with antibiotic ointment
C) Wet to dry dressing
D) Occlusive moist dressing
D) Occlusive moist dressing

A 30 month-old child is admitted to the hospital unit. Which of the following toys would be appropriate for the nurse to select from the toy room for this child?
A) Cartoon stickers
B) Large wooden puzzle
C) Blunt scissors and paper
D) Beach ball
B) Large wooden puzzle

A nurse is to present information about Chinese folk medicine to a group of student
nurses. Based on this cultural belief, the nurse would explain that illness is attributed to the
A) Yang, the positive force that represents light, warmth, and fullness
B) Yin, the negative force that represents darkness, cold, and emptiness
C) Use of improper hot foods, herbs and plants
D) A failure to keep life in balance with nature and others
B) Yin, the negative force that represents darkness, cold, and emptiness

A 2 year-old child has just been diagnosed with cystic fibrosis. The child’s father asks the nurse “What is our major concern now, and what will we have to deal with in the future?” Which of the following is the best response?
A) “There is a probability of life-long complications.”
B) “Cystic fibrosis results in nutritional concerns that can be dealt with.”
C) “Thin, tenacious secretions from the lungs are a constant struggle in cystic fibrosis.”
D) “You will work with a team of experts and also have access to a support group that the family can attend.”
C) “Thin, tenacious secretions from the lungs are a constant struggle in cystic fibrosis.”

Which type of accidental poisoning would the nurse expect to occur in children under
age 6?
A) Oral ingestion
B) Topical contact
C) Inhalation
D) Eye splashes
A) Oral ingestion

A client was admitted to the psychiatric unit with a diagnosis of bipolar disorder. He constantly bothers other clients, tries to help the housekeeping staff, demonstrates pressured speech and demands constant attention from the staff. Which activity would be best for the client?
A) Reading
B) Checkers
C) Cards
D) Ping-pong
D) Ping-pong

The nurse is caring for a client who has developed cardiac tamponade. Which finding
would the nurse anticipate?
A) Widening pulse pressure
B) Pleural friction rub
C) Distended neck veins
D) Bradycardia
C) Distended neck veins

Which nursing action is a priority as the plan of care is developed for a 7 year-old
child hospitalized for acute glomerulonephritis?
A) Assess for generalized edema
B) Monitor for increased urinary output
C) Encourage rest during hyperactive periods
D) Note patterns of increased blood pressure
D) Note patterns of increased blood pressure

The nurse is caring for a child receiving chest physiotherapy (CPT). Which of the
following actions by the nurse would be appropriate?
A) Schedule the therapy thirty minutes after meals
B) Teach the child not to cough during the treatment
C) Confine the percussion to the rib cage area
D) Place the child in a prone position for the therapy
C) Confine the percussion to the rib cage area

Why is it important for the nurse to monitor blood pressure in clients receiving antipsychotic drugs?
A) Orthostatic hypotension is a common side effect
B) Most antipsychotic drugs cause elevated blood pressure
C) This provides information on the amount of sodium allowed in the diet
D) It will indicate the need to institute anti parkinsonian drugs
A) Orthostatic hypotension is a common side effect

The nurse is teaching the client to select foods rich in potassium to help prevent digitalis toxicity. Which choice indicates the client understands dietary needs?
A) Three apricots
B) Medium banana
C) Naval orange
D) Baked potato
D) Baked potato

An 86 year-old nursing home resident who has decreased mental status is hospitalized with pneumonic infiltrates in the right lower lobe. When the nurse assists the client with a clear liquid diet, the client begins to cough. What should the nurse do next?
A) Add a thickening agent to the fluids
B) Check the client’s gag reflex
C) Feed the client only solid foods
D) Increase the rate of intravenous fluids
B) Check the client’s gag reflex

The nurse is planning care for a client with a CVA. Which of the following measures planned by the nurse would be most effective in preventing skin breakdown?
A) Place client in the wheelchair for four hours each day
B) Pad the bony prominence
C) Reposition every two hours
D) Massage reddened bony prominence
C) Reposition every two hours

A nurse is assessing several clients in a long term health care facility. Which client is at highest risk for development of decubitus ulcers?
A) A 79 year-old malnourished client on bed rest
B) An obese client who uses a wheelchair
C) A client who had 3 incontinent diarrhea stools
D) An 80 year-old ambulatory diabetic client
C) A client who had 3 incontinent diarrhea stools

Constipation is one of the most frequent complaints of elders. When assessing this problem, which action should be the nurse’s priority?
A) Obtain a complete blood count
B) Obtain a health and dietary history
C) Refer to a provider for a physical examination
D) Measure height and weight
B) Obtain a health and dietary history

After a client has an enteral feeding tube inserted, the most accurate method for verification of placement is
A) Abdominal x-ray
B) Auscultation
C) Flushing tube with saline
D) Aspiration for gastric contents
A) Abdominal x-ray

A client was just taken off the ventilator after surgery and has a nasogastric tube draining bile colored liquids. Which nursing measure will provide the most comfort to the client?A) Allow the client to melt ice chips in the mouth
B) Provide mints to freshen the breath
C) Perform frequent oral care with a tooth sponge
D) Swab the mouth with glycerin swabs
C) Perform frequent oral care with a tooth sponge

The nurse is instructing a 65 year-old female client diagnosed with osteoporosis. The most important instruction regarding exercise would be to
A) Exercise doing weight bearing activities
B) Exercise to reduce weight
C) Avoid exercise activities that increase the risk of fracture D) Exercise to strengthen muscles and thereby protect bones
A) Exercise doing weight bearing activities

The nurse has been teaching a client with congestive heart failure about proper nutrition. The selection of which lunch indicates the client has learned about sodium restriction?
A) Cheese sandwich with a glass of 2% milk
B) Sliced turkey sandwich and canned pineapple
C) Cheeseburger and baked potato
D) Mushroom pizza and ice cream
B) Sliced turkey sandwich and canned pineapple

Which bed position is preferred for use with a client in an extended care facility on falls risk prevention protocol?
A) All 4 side rails up, wheels locked, bed closest to door
B) Lower side rails up, bed facing doorway
C) Knees bent, head slightly elevated, bed in lowest position
D) Bed in lowest position, wheels locked, place bed against wall
D) Bed in lowest position, wheels locked, place bed against wall

The nurse is talking to parents about nutrition in school aged children. Which of the following is the most common nutritional disorder in this age group?
A) Bulimia
B) Anorexia
C) Obesity
D) Malnutrition
C) Obesity

At the geriatric day care program a client is crying and repeating “I want to go home. Call my daddy to come for me.” The nurse should
A) Invite the client to join the exercise group
B) Tell the client you will call someone to come for her
C) Give the client simple information about what she will be doing
D) Firmly direct the client to her assigned group activity
C) Give the client simple information about what she will be doing

A victim of domestic violence states to the nurse, “If only I could change and be how my companion wants me to be, I know things would be different.” Which would be the best response by the nurse?
A) “The violence is temporarily caused by unusual circumstances, don’t stop hoping for a change.
“B) “Perhaps, if you understood the need to abuse, you could stop the violence.
“C) “No one deserves to be beaten. Are you doing anything to provoke your spouse into beating you?”
D) “Batterers lose self-control because of their own internal reasons, not because of what their
partner did or did not do.”
D) “Batterers lose self-control because of their own internal reasons, not because of what their
partner did or did not do.”

A 38 year-old female client is admitted to the hospital with an acute exacerbation of asthma. This is her third admission for asthma in 7 months. She describes how she doesn’t really like having to use her medications all the time. Which explanation by the nurse best describes the long-term consequence of uncontrolled airway inflammation?
A) Degeneration of the alveoli
B) Chronic broncho constriction of the large airways
C) Lung remodeling and permanent changes in lung function
D) Frequent pneumonia
C) Lung remodeling and permanent changes in lung function

A mother wants to switch her 9 month-old infant from an iron fortified formula to whole milk because of the expense. Upon further assessment, the nurse finds that the baby eats table foods well, but drinks less milk than before. What is the best advice by the nurse?
A) Change the baby to whole milk
B) Add chocolate syrup to the bottle
C) Continue with the present formula
D) Offer fruit juice frequently
C) Continue with the present formula

Privacy and confidentiality of all client information is legally protected. In which of
these situations would the nurse make an exception to this practice?
A) When a family member offers information about their loved one
B) When the client threatens self-harm and harm to others
C) When the health care provider decides the family has a right to know the client’s diagnosis
D) When a visitor insists that the visitor has been given permission by the client
B) When the client threatens self-harm and harm to others

The nurse is caring for a client who is in the late stage of multiple myeloma. Which of
the following should be included in the plan of care?
A) Monitor for hyperkalemia
B) Place in protective isolation
C) Precautions with position changes
D) Administer diuretics as ordered
C) Precautions with position changes

The nurse is making a home visit to a client with chronic obstructive pulmonary disease (COPD). The client tells the nurse that he used to be able to walk from the house to the mailbox without difficulty. Now, he has to pause to catch his breath halfway through the trip. Which diagnosis would be most appropriate for this client based on this assessment?
A) Activity intolerance caused by fatigue related to chronic tissue hypoxia
B) Impaired mobility related to chronic obstructive pulmonary disease
C) Self-care deficit caused by fatigue related to dyspnea
D) Ineffective airway clearance related to increased bronchial secretions
A) Activity intolerance caused by fatigue related to chronic tissue hypoxia

The nurse admits a client newly diagnosed with hypertension. What is the best
method for assessing the blood pressure?
A) Standing and sitting
B) In both arms
C) After exercising
D) Supine position
B) In both arms

The nurse is caring for residents in a long term care setting for the elderly. Which of the following activities will be most effective in meeting the growth and development needs for persons in this age group?
A) Aerobic exercise classes
B) Transportation for shopping trips
C) Reminiscence groups
D) Regularly scheduled social activities
C) Reminiscence groups

Post-procedure nursing interventions for electroconvulsive therapy include
A) Applying hard restraints if seizure occurs
B) Expecting client to sleep for 4 to 6 hours
C) Remaining with client until oriented
D) Expecting long-term memory loss
C) Remaining with client until oriented

The nurse assesses delayed gross motor development in a 3 year-old child. The
inability of the child to do which action confirms this finding?
A) Stand on 1 foot
B) Catch a ball
C) Skip on alternate feet
D) Ride a bicycle
A) Stand on 1 foot

The mother of a 15 month-old child asks the nurse to explain her child’s lab results
and how they show her child has iron deficiency anemia. The nurse’s best response is
A) “Although the results are here, your doctor will explain them later.
“B) “Your child has less red blood cells that carry oxygen.
“C) “The blood cells that carry nutrients to the cells are too large.”
D) “There are not enough blood cells in your child’s circulation.”
B) “Your child has less red blood cells that carry oxygen.”

In a child with suspected coarctation of the aorta, the nurse would expect to find
A) Strong pedal pulses
B) Diminishing carotid pulses
C) Normal femoral pulses
D) Bounding pulses in the arms
D) Bounding pulses in the arms

At the day treatment center a client diagnosed with Schizophrenia – Paranoid Type sits alone alertly watching the activities of clients and staff. The client is hostile when approached and asserts that the doctor gives her medication to control her mind. The client’s behavior most likely indicates
A) Feelings of increasing anxiety related to paranoia
B) Social isolation related to altered thought processes
C) Sensory perceptual alteration related to withdrawal from environment
D) Impaired verbal communication related to impaired judgment
B) Social isolation related to altered thought processes

A 65-year-old Hispanic-Latino client with prostate cancer rates his pain as a 6 on a 0- to-10 scale. The client refuses all pain medication other than Motrin, which does not relieve his pain. The next action for the nurse to take is to
A) Ask the client about the refusal of certain pain medications
B) Talk with the client’s family about the situation
C) Report the situation to the health care provider
D) Document the situation in the notes
A) Ask the client about the refusal of certain pain medications

What nursing assessment of a paralyzed client would indicate the probable presence of a fecal impaction?
A) Presence of blood in stools
B) Oozing liquid stool
C) Continuous rumbling flatulence
D) Absence of bowel movements
B) Oozing liquid stool

A client in a long term care facility complains of pain. The nurse collects data about the client’s pain. The first step in pain assessment is for the nurse to
A) Have the client identify coping methods
B) Get the description of the location and intensity of the pain
C) Accept the client’s report of pain
D) Determine the client’s status of pain
C) Accept the client’s report of pain

An 85 year-old client complains of generalized muscle aches and pains. The first action by the nurse should be
A) Assess the severity and location of the pain
B) Obtain an order for an analgesic
C) Reassure him that this is not unusual for his age
D) Encourage him to increase his activity
A) Assess the severity and location of the pain

A 20 year-old client has an infected leg wound from a motorcycle accident, and the client has returned home from the hospital. The client is to keep the affected leg elevated and is on contact precautions. The client wants to know if visitors can come. The appropriate response from the home health nurse is that:
A) Visitors must wear a mask and a gown
B) There are no special requirements for visitors of clients on contact precautions
C) Visitors should wash their hands before and after touching the client
D) Visitors
C) Visitors should wash their hands before and after touching the client

A child is admitted to the pediatric unit with a diagnosis of suspected meningococcal meningitis. Which admission orders should the nurse do first?
A) Institute seizure precautions
B) Monitor neurologic status every hour
C) Place in respiratory/secretion precautions
D) Cefotaxime IV 50 mg/kg/day divided q6h
C) Place in respiratory/secretion precautions

Which of these nursing diagnoses of 4 elderly clients would place 1 client at the greatest risk for falls?
A) Sensory perceptual alterations related to decreased vision
B) Alteration in mobility related to fatigue
C) Impaired gas exchange related to retained secretions
D) Altered patterns of urinary elimination related to nocturia
D) Altered patterns of urinary elimination related to nocturia

A newly admitted adult client has a diagnosis of hepatitis A. The charge nurse should reinforce to the staff members that the most significant routine infection control strategy, in addition to hand washing, to be implemented is which of these?
A) Apply appropriate signs outside and inside the room
B) Apply a mask with a shield if there is a risk of fluid splash C) Wear a gown to change soiled linens from incontinence
D) Have gloves on while handling bedpans with feces
D) Have gloves on while handling bedpans with feces

Which of these clients with associated lab reports is a priority for the nurse to report to the public health department within the next 24 hours?
A) An infant with a positive culture of stool for Shigella
B) An elderly factory worker with a lab report that is positive for acid-fast bacillus smear
C) A young adult commercial pilot with a positive histopathological examination from an induced sputum for Pneumocystis carinii
D) A middle-aged nurse with a history of varicella-zoster virus and with crops of vesicles on an erythematous base that appear on the skin
B) An elderly factory worker with a lab report that is positive for acid-fast bacillus smear

A client is diagnosed with methicillin resistant staphylococcus aureus pneumonia. What type of isolation is most appropriate for this client?
A) Reverse
B) Airborne
C) Standard precautions
D) Contact
D) Contact

The school nurse is teaching the faculty the most effective methods to prevent the spread of lice in the school. The information that would be most important to include would be which of these statements?
A) “The treatment requires reapplication in 8 to 10 days.”
B) “Bedding and clothing can be boiled or steamed.”
C) Children are not to share hats, scarves and combs.
D) Nit combs are necessary to comb out nits.
C) Children are not to share hats, scarves and combs.

During the care of a client with a salmonella infection, the primary nursing intervention to limit transmission is which of these approaches?
A) Wash hands thoroughly before and after client contact
B) Wear gloves when in contact with body secretions
C) Double glove when in contact with feces or vomitus
D) Wear gloves when disposing of contaminated linens
A) Wash hands thoroughly before and after client contact

A nurse is reinforcing teaching with a client about compromised host precautions. The client is receiving filgrastim (Neupogen) for neutropenia. The selection of which lunch suggests the client has learned about necessary dietary changes?
A) Grilled chicken sandwich and skim milk
B) Roast beef, mashed potatoes, and green beans
C) Peanut butter sandwich, banana, and iced tea
D) Barbecue beef, baked beans, and cole slaw
B) Roast beef, mashed potatoes, and green beans

After talking with her partner, a client voluntarily admitted herself to the substance abuse unit. After the second day on the unit the client states to the nurse, “My husband told me to get treatment or he would divorce me. I don’t believe I really need treatment but I don’t want my husband to leave me.” Which response by the nurse would assist the client?
A) “In early recovery, it’s quite common to have mixed feelings, but unmotivated people can’t get well.”
B) “In early recovery, it’s quite common to have mixed feelings, but I didn’t know you had been pressured to come.”
C) “In early recovery it’s quite common to have mixed feelings, perhaps it would be best to seek treatment on an out client bases. “
D) ” In early recovery, it’s quite common to have mixed feelings. Let’s discuss the benefits of sobriety for you.”
D) ” In early recovery, it’s quite common to have mixed feelings. Let’s discuss the benefits of sobriety for you.”

A neonate born 12 hours ago to a methadone maintained woman is exhibiting a hyperactive MORO reflex and slight tremors. The newborn passes loose, watery stool. Which of these is a nursing priority?
A) Hold the infant at frequent intervals.
B) Assess for neonatal withdrawal syndrome
C) Offer fluids to prevent dehydration
D) Administer paregoric to stop diarrhea
B) Assess for neonatal withdrawal syndrome

The nurse is caring for a post myocardial infarction client in an intensive care unit. It is noted that urinary output has dropped from 60 -70 ml per hour to 30 ml per hour. This change is most likely due to
A) Dehydration
B) Diminished blood volume
C) Decreased cardiac output
D) Renal failure
C) Decreased cardiac output

The primary nursing diagnosis for a client with congestive heart failure with pulmonary edema is
A) Pain
B) Impaired gas exchange
C) Cardiac output altered: decreased
D) Fluid volume excess
C) Cardiac output altered: decreased

The nurse is performing a developmental assessment on an 8 month-old. Which finding should be reported to the health care provider?
A) Lifts head from the prone position
B) Rolls from abdomen to back
C) Responds to parents’ voices
D) Falls forward when sitting
D) Falls forward when sitting

A client has received her first dose of fluphenazine (Prolixin) 2 hours ago. She suddenly experiences torticollis and involuntary spastic muscle movement. In addition to administering the ordered anticholinergic drug, what other measure should the nurse implement?
A) Have respiratory support equipment available
B) Immediately place her in the seclusion room
C) Assess the client for anxiety and agitation
D) Administer PRN dose of IM antipsychotic medication
A) Have respiratory support equipment available

The nurse walks into a client’s room and finds the client lying still and silent on the floor. The nurse should first
A) Assess the client’s airway
B) Call for help
C) Establish that the client is unresponsive
D) See if anyone saw the client fall
C) Establish that the client is unresponsive

The nurse is caring for a client 2 hours after a right lower lobectomy. During the evaluation of the water-seal chest drainage system, it is noted that the fluid level bubbles constantly in the water seal chamber. On inspection of the chest dressing and tubing, the nurse does not find any air leaks in the system. The next best action for the nurse is to
A) Check for subcutaneous emphysema in the upper torso
B) Reposition the client to a position of comfort
C) Call the health care provider as soon as possible
D) Check for any increase in the amount of thoracic drainage
A) Check for subcutaneous emphysema in the upper torso

The nurse is teaching a client with dysrhythmia about the electrical pathway of an impulse as it travels through the heart. Which of these demonstrates the normal pathway?
A) AV node, SA node, Bundle of His, Purkinje fibers
B) Purkinje fibers, SA node, AV node, Bundle of His
C) Bundle of His, Purkinje fibers, SA node , AV node
D) SA node, AV node, Bundle of His, Purkinje fibers
D) SA node, AV node, Bundle of His, Purkinje fibers

When assessing a client who has just undergone a cardioversion, the nurse finds the respirations are 12. Which action should the nurse take first?
A) Try to vigorously stimulate normal breathing
B) Ask the RN to assess the vital signs
C) Measure the pulse oximetry
D) Continue to monitor respirations
D) Continue to monitor respirations

When assessing a client, it is important for the nurse to be informed about cultural issues related to the client’s background because
A) Normal patterns of behavior may be labeled as deviant, immoral, or insane
B) The meaning of the client’s behavior can be derived from conventional wisdom
C) Personal values will guide the interaction between persons from 2 cultures
D) The nurse should rely on her knowledge of different developmental mental stages
A) Normal patterns of behavior may be labeled as deviant, immoral, or insane

The nurse is responsible for several elderly clients, including a client on bed rest with a skin tear and hematoma from a fall 2 days ago. What is the best care assignment for this client?
A) Assign an RN to provide total care of the client
B) Assign a nursing assistant to help the client with self-care activities
C) Delegate complete care to an unlicensed assistive personnel
D) Supervise a nursing assistant for skin care
D) Supervise a nursing assistant for skin care

The nursing student is discussing with a preceptor the delegation of tasks to an unlicensed assistive personnel (UAP). Which tasks, delegated to a UAP, indicates the student needs further teaching about the delegation process?
A) Assist a client post cerebral vascular accident to ambulate
B) Feed a 2 year-old in balanced skeletal traction
C) Care for a client with discharge orders
D) Collect a sputum specimen for acid fast bacillus
C) Care for a client with discharge orders

After working with a very demanding client, an unlicensed assistive personnel(UAP) tells the nurse, “I have had it with that client. I just can’t do anything that pleases him. I’m not going in there again.” The nurse should respond by saying
A) “He has a lot of problems. You need to have patience with him.
“B) “I will talk with him and try to figure out what to do.”
C) “He is scared and taking it out on you. Let’s talk to figure out what to do.”
D) “Ignore him and get the rest of your work done. Someone else can take care of him forthe rest of the day.”
C) “He is scared and taking it out on you. Let’s talk to figure out what to do.”

A client with a diagnosis of bipolar disorder has been referred to a local boarding home for consideration for placement. The social worker telephoned the hospital unit for information about the client’s mental status and adjustment. The appropriate response of the nurse should be which of these statements?
A) I am sorry. Referral information can only be provided by the client’s health care providers.
B) “I can never give any information out by telephone. How do I know who you are?”
C) Since this is a referral, I can give you this information.
D) I need to get the client’s written consent before I release any information to you.
D) I need to get the client’s written consent before I release any information to you.

A client is admitted with a diagnosis of schizophrenia. The client refuses to take medication and states “I don’t think I need those medications. They make me too sleepy and drowsy. I insist that you explain their use and side effects.” The nurse should understand that
A) A referral is needed to the psychiatrist who is to provide the client with answers
B) The client has a right to know about the prescribed medications
C) Such education is an independent decision of the individual nurse whether or not to teach clients about their medications
D) Clients with schizophrenia are at a higher risk of psychosocial complications when they know about their medication side effects
B) The client has a right to know about the prescribed medications

Which statement by the nurse is appropriate when asking an unlicensed assistive personnel (UAP) to assist a 69 year-old surgical client to ambulate for the first time?
A) “Have the client sit on the side of the bed for at least 2 minutes before helping him stand.”
B) “If the client is dizzy on standing, ask him to take some deep breaths.”
C) “Assist the client to the bathroom at least twice on this shift.”
D) “After you assist him to the chair, let me know how he feels.”
A) “Have the client sit on the side of the bed for at least 2 minutes before helping him stand.”

The nurse receives a report on an older adult client with middle stage dementia.What information suggests the nurse should do immediate follow up rather than delegate care to the nursing assistant? The client
A) Has had a change in respiratory rate by an increase of 2 breaths
B) Has had a change in heart rate by an increase of 10 beats
C) Was minimally responsive to voice and touch
D) Has had a blood pressure change by a drop in 8 mmHg systolic
C) Was minimally responsive to voice and touch

A client tells the nurse, “I have something very important to tell you if you promise not to tell.” The best response by the nurse is
A) “I must document and report any information.”
B) “I can’t make such a promise.”
C) “That depends on what you tell me.”
D) “I must report everything to the treatment team.”
C) Was minimally responsive to voice and touch

Which task could be safely delegated by the nurse to an unlicensed assistive personnel (UAP)?
A) Be with a client who self-administers insulin
B) Cleanse and dress a small decubitus ulcer
C) Monitor a client’s response to passive range of motion exercises
D) Apply and care for a client’s rectal pouch
D) Apply and care for a client’s rectal pouch

A client asks the nurse to call the police and states: “I need to report that I am being abused by a nurse.” The nurse should first
A) Focus on reality orientation to place and person
B) Assist with the report of the client’s complaint to the police
C) Obtain more details of the client’s claim of abuse
D) Document the statement on the client’s chart with a report to the manager
C) Obtain more details of the client’s claim of abuse

A nurse from the maternity unit is floated to the critical care unit because of staff shortage on the evening shift. Which client would be appropriate to assign to this nurse? A client with
A) A Dopamine drip IV with vital signs monitored every 5 minutes
B) A myocardial infarction that is free from pain and dysrhythmias
C) A tracheotomy of 24 hours in some respiratory distress
D) A pacemaker inserted this morning with intermittent capture
B) A myocardial infarction that is free from pain and dysrhythmias

An unlicensed assistive personnel (UAP), who usually works on a surgical unit is assigned to float to a pediatric unit. Which question by the charge nurse would be most appropriate when making delegation decisions?
A) “How long have you been a UAP and what units you have worked on?”
B) “What type of care do you give on the surgical unit and what ages of clients?”
C) “What is your comfort level in caring for children and at what ages?”
D) “Have you reviewed the list of expected skills you might need on this unit?”
D) “Have you reviewed the list of expected skills you might need on this unit?”

A client frequently admitted to the locked psychiatric unit repeatedly compliments and invites one of the nurses to go out on a date. The nurse’s response should be to
A) Ask to not be assigned to this client or to work on another unit
B) Tell the client that such behavior is inappropriate
C) Inform the client that hospital policy prohibits staff to date clients
D) Discuss the boundaries of the therapeutic relationship with the client
D) Discuss the boundaries of the therapeutic relationship with the client

In order to enhance a client’s response to medication for chest pain from acute angina, the nurse should emphasize
A) Learning relaxation techniques
B) Limiting alcohol use
C) Eating smaller meals
D) Avoiding passive smoke
A) Learning relaxation techniques

The nurse is caring for 2 children who have had surgical repair of congenital heart defects. For which defect is it a priority to assess for findings of heart conduction disturbance?
A) Arterial septal defect
B) Patent ductus arteriosus
C) Aortic stenosis
D) Ventricular septal defect
D) Ventricular septal defect

Clients with mitral stenosis would likely manifest findings associated with congestion in the
A) Pulmonary circulation
B) Descending aorta
C) Superior vena cava
D) Bundle of His
A) Pulmonary circulation

The nurse is teaching a smoking cessation class and notices there are 2 pregnant women in the group. Which information is a priority for these women?
A) Low tar cigarettes are less harmful during pregnancy
B) There is a relationship between smoking and low birth weight
C) The placenta serves as a barrier to nicotine
D) Moderate smoking is effective in weight control
B) There is a relationship between smoking and low birth weight

What is the best way for the nurse to accomplish a health history on a 14 year-old client?
A) Have the mother present to verify information
B) Allow an opportunity for the teen to express feelings
C) Use the same type of language as the adolescent
D) Focus the discussion of risk factors in the peer group
B) Allow an opportunity for the teen to express feelings

We have an expert-written solution to this problem!
What principle of HIV disease should the nurse keep in mind when planning care for a newborn who was infected in utero?
A) The disease will incubate longer and progress more slowly in this infant
B) The infant is very susceptible to infections
C) Growth and development patterns will proceed at a normal rate
D) Careful monitoring of renal function is indicated
B) The infant is very susceptible to infections

While planning care for a preschool aged child, the nurse understands needs. Which of the following would be of the most concern to the nurse?
A) Playing imaginatively
B) Expressing shame
C) Identifying with family
D) Exploring the playroom
B) Expressing shame

A client has been receiving lithium (Lithane) for the past two weeks for the treatment of bipolar illness. When planning client teaching, what is most important to emphasize to the client?
A) Maintain a low sodium diet
B) Take a diuretic with lithium
C) Come in for evaluation of serum lithium levels every 1-3 months
D) Have blood lithium levels drawn during the summer months
D) Have blood lithium levels drawn during the summer months

While teaching a client about their medications, the client asks how long it will take before the effects of lithium take place. What is the best response of the nurse?
A) Immediately
B) Several days
C) 2 weeks
D) 1 month
C) 2 weeks

The nursing intervention that best describes treatment to deal with the behaviors of clients with personality disorders include
A) Pointing out inconsistencies in speech patterns to correct thought disorders
B) Accepting client and the client’s behavior unconditionally
C) Encouraging dependency in order to develop ego controls
D) Consistent limit-setting enforced 24 hours per day
D) Consistent limit-setting enforced 24 hours per day

Following a cocaine high, the user commonly experiences an extremely unpleasant feeling called
A) Craving
B) Crashing
C) Outward bound
D) Nodding out
B) Crashing

The nurse asks a client with a history of alcoholism about the client’s drinking behavior. The client states “I didn’t hurt anyone. I just like to have a good time, and drinking helps me to relax.” The client is using which defense mechanism?
A) Denial
B) Projection
C) Intellectualization
D) Rationalization
D) Rationalization

One reason that domestic violence remains extensively undetected is
A) Few battered victims seek medical care
B) There is typically a series of minor, vague complaints
C) Expenses due to police and court costs are prohibitive
D) Very little knowledge is currently known about batterers and battering relationships
B) There is typically a series of minor, vague complaints

A client develops volume overload from an IV that has infused too rapidly. What assessment would the nurse expect to find?
A) S3 heart sound
B) Thready pulse
C) Flattened neck veins
D) Hypoventilation
A) S3 heart sound

An explosion has occurred at a high school for children with special needs and severe developmental delays. One of the students accompanied with a parent is seen at a community health center a day later. After the initial assessment the nurse concludes that the student appears to be in a crisis state. Which of these interventions based on crisis intervention principles is appropriate to do next?
A) Help the student to identify a specific problem
B) Ask the parent to identify the major problem
C) Ask the student to think of different alternatives
D) Examine with the parent a variety of options
B) Ask the parent to identify the major problem

Which statement made by a client to the admitting nurse suggests that the client is experiencing a manic episode?
A) “I think all children should have their heads shaved.
“B) “I have been restricted in thought and harmed.”
C) “I have powers to get you whatever you wish, no matter the cost.
” D) “I think all of my contacts last week have attempted to poison me.”
C) “I have powers to get you whatever you wish, no matter the cost.”

A client says, “It’s raining outside and it’s raining in my heart. Did you know that St. Patrick drove the snakes out of Ireland? I’ve never been to Ireland.” The nurse would document this behavior as
A) Perseveration
B) Circumstantiality
C) Neologisms
D) Flight of ideas
D) Flight of ideas

During the change-of-shift report the assigned nurse notes a Catholic client is scheduled to be admitted for the delivery of a ninth child. Which comment stated angrily to a colleague by this nurse indicates an attitude of prejudice?
A) “I wonder who is paying for this trip to the hospital?
“B) “I think she needs to go to the city hospital.
“C) “All those people indulge in large families!
“D) “Doesn’t she know there’s such a thing as birth control?”
D) “Doesn’t she know there’s such a thing as birth control?”

Which of these statements by the nurse reflects the best use of therapeutic interaction techniques?
A) “You look upset. Would you like to talk about it?
“B) “I’d like to know more about your family. Tell me about them.”
C) “I understand that you lost your partner. I don’t think I could go on if that happened to me.”
D) “You look very sad. How long have you been this way?”
A) “You look upset. Would you like to talk about it?”

A nurse in the emergency department suspects domestic violence as the cause of a client’s injuries. What action should the nurse take first?
A) Ask client if there are any old injuries also present
B) Interview the client without the persons who came with the client
C) Gain client’s trust by not being hurried during the intake process
D) Photograph the specific injuries in question
B) Interview the client without the persons who came with the client

A morbidly obese client is scheduled for gastric bypass surgery. The client completes the required preoperative nutritional counseling and signs the operative permit. To promote effective discharge planning, which intervention is most important for the nurse to implement?
Discuss small, low fat, low sugar meal preparation techniques.

A client who was splashed with a chemical has both eyes covered with bandages. When assisting the client with eating, which intervention should the nurse instruct the UAP to implement?
Orient the client to the location of the food on the plate.

A client is admitted for an exacerbation of heart failure (HF) and is being treated with diuretics for fluid volume excess. In planning nursing care, which interventions should the nurse include? (Select all that apply.)

  • Observe for evidence of hypokalemia
  • Teach the client how to restrict dietary sodium
  • Weight the client daily, in the morning

While changing the dressing of a client who is immobile, the nurse observes a red and swollen wound with a moderate amount of yellow and green drainage and a foul odor. Given there is a positive methicillin-resistant Staphylococcus aureus (MRSA), which is the most important action for the nurse to take?
Administer prescribed antibiotics.

A client who has asthma receives a new prescription for a corticosteroid inhaler. The client expresses concern about taking steroid medications. What information should the nurse provide the client about the use of this maintenance inhaler?
Systemic side effects are reduced when taken by inhalation

The healthcare provider prescribes a sedative for a client with severe hypothyroidism. The nurse plans to contact the provider to review the safety of the prescription for the client and consults first with the charge nurse. The charge nurse notes that the prescription is written legally and completely. How should the charge nurse respond?
Assume responsibility for discussing the concern with the provider.

In assessing a client twelve hours following transurethral resection of the prostate (TURP), the nurse observes that the urinary drainage tubing contains a large amount of clear pale pink urine and the continuous bladder irrigation is infusing slowly. Which action should the nurse implement?
Ensure that no dependent loops are present in the tubing.

Which information should the nurse include in the discharge teaching plan of a client with low back pain who is taking cyclobenzaprine to control muscle spasms?
Use cold and allergy medications only as directed by a healthcare provider.

A client with deep vein thrombosis (DVT) is receiving a continuous intravenous heparin infusion. The client now has tarry, black diarrhea and reports abdominal pain. Which actions should the nurse implement? (Select all that apply)

  • Monitor stools for presence of blood
  • Auscultate bowel sounds in all quadrants
  • Review last partial thromboplastin time results.

Which laboratory results should the nurse closely monitor in a client who has end-stage renal disease (ESRD)?

  • Serum potassium, calcium, and phosphorus.

A mal client who was in a motor vehicle collision yesterday is receiving a unit of packed red blood cells. When half of the unit is infused, the client reports lower back pain, and the nurse observes a fine rash over his chest and back. Which intervention should the nurse implement?
Replace the transfusion with normal saline

When developing a teaching plan for a client with newly diagnosed Type 1 diabetes, the nurse should explain that an increased thirst is an early sign of diabetic ketoacidosis (DKA). Which action should the nurse instruct the client to implement if this sign of DKA occurs?
Give a dose of regular insulin as prescribed

Which information is most important to include in the teaching plan for a client who is discharged after a thyroidectomy for Grave’s disease?
Daily hormone replacement will be needed for the rest of the client’s life.

The nurse is planning discharge instructions for a client with type 2 diabetes who will be starting exenatide. Which information should be included in the discharge instructions?
Notify your healthcare provider if you start having abdominal pain.

Which breakfast selection should he nurse recommend for a 16-year-old with diarrhea?
Oatmeal, banana, and herbal tea.

The nurse is preparing to administer 1.6 mL of medication intramuscularly to a 4-month-old infant. Which action should the nurse include?
Divide the medication into two injections with volumes under 1 mL.

A school-aged child with chronic renal failure receives a prescription from the healthcare provider for losartan. Which action should the nurse implement prior to administering the medication?
Evaluate the ability to swallow medication.

An adolescent client on a drug treatment unit becomes angry and pulls the refrigerator from the wall and then throws the microwave. After the client fails to respond to the redirection, the healthcare provider prescribes restraints. Which assessment should the nurse include in the client’s record while the client is in restraints?

  • Range-of-motion and circulation.

The nurse brings an oral medication prescribed to be given daily to a male client who tells the nurse that he will take his medication later. Which action should the nurse implement?
Agree upon a time to return to the client’s room with the medication.

When conducting diet teaching for a client who was diagnosed with hypertension, which foods should the nurse encourage the client to eat? (Select all that apply)

  • Fresh or frozen vegetables without sauce
  • Fruits without sauce

The charge nurse observes a new nurse preparing to insert an intravenous (IV) catheter. The new nurse gathers equipment, including an intravenous catheter and insertion kit, and a 4×4 gauze dressing to apply over the insertion site. Which action should the charge nurse take?

  • Remind the nurse to use a transparent dressing over the site.

The client who has emphysema and recently experienced a stroke receives a prescription for a metered-dose inhaler with a spacer device. The client asks the nurse if using the spacer is necessary. What information should the nurse provide?
It allows time to inhale the entire dispensed dose.

A client with atrial fibrillation receives a new prescription for dabigatran etexilate. Which instruction is important for the nurse to emphasize when teaching the client about this medication?

  • Report unusual bruising or bleeding.

An older adult with a terminal illness is receiving hospice care and is having difficulty coping with feelings related to death and dying. Which interventions should the nurse include in this client’s plan of care?

  • Encourage family to bring the client old photographs
  • Encourage family to visit frequently
  • Teach the client how to use guided imagery

The nurse is preparing an older client for discharge following cataract extraction. Which instruction should be included in the discharge teaching?
Avoid straining at school, bending, or lifting heavy objects.

The nurse completes percussion of the abdomen on an older adult client. Which finding is considered normal for this client?
Musical and drumlike.

An older client is admitted for repair of a broken hip. To reduce the risk for infection in the postoperative period, which nursing care interventions should the nurse include in the client’s plan of care?

  • Teach the client to use incentive spirometer every 2 hours while awake.
  • Remove urinary catheter as soon as possible and encourage voiding.

An older adult ids admitted to the stroke unit after recovery from the acute phrase of an ischemic cerebral vascular accident (CVA). Which interventions should the nurse include in the plan of care during convalescence and rehabilitation?

  • Place a bedside commode next to bed.
  • Encourage family to participate in the client’s care.
  • Measure neurological vital signs every 4 hours.

The nurse is triaging victims of an explosion from a housing area outside of town. The nurse should issue a black disaster tag to which client?

  • An older client with a head injury, fixed pupils and absent vital signs.

A client in labor begins bleeding profusely from the vagina. Which findings should the nurse report to the healthcare provider?
Sharp fundal pain and uterine tenderness.

A client is admitted to the labor and delivery unit in early labor and the nurse assesses the status of her contractions. The frequency of contractions is most accurately by counting the minutes and seconds in which manner?
From the beginning of one contraction to the beginning of the next contraction.

A postterm infant is delivered with meconium-stained skin and cord. The newborn is having substernal retractions, grunting, and nasal flaring. Surfactant is administered. Which statement should the nurse provide to the family about the purpose of surfactant?
It increases lung compliance and decreases surface tension.

A postpartum client who is bottle feeding develops breast engorgement. What is the best recommendation for the nurse to provide this client?
Avoid stimulation of the breasts and wear a tight bra.

The nurse finds a client at 33-week gestation in cardiac arrest. What adaptation to cardiopulmonary resuscitation (CPR) should the nurse implement?
Position a firm wedge to support pelvis and thorax at 30 degree tilt.

Six weeks after the birth of child with Trisomy 21, the parents return to the prenatal clinic for a follow-up visit. They have spoken with a genetic counselor, but are still unsure about the risk of having another clid with Trisomy 21. The couple begins literature form the counselor with them, and asks the nurse to explain it. Which actions should the nurse take?
Review the literature and answer any questions the nurse is able to answer.

A client hospitalized with pleural effusion has a history of heart failure. To reduce cardiac workload, which intervention should the nurse include in the client’s plan of care?
Provide a bedside commode for toileting

Two hours after an abdominal aneurysm repair, a client remains sedated and mechanically ventilated in the post-anesthesia care unit (PACU). Which assessment finding warrants immediate intervention by the nurse?
Systolic blood pressure less than 90 mm Hg.

An older adult male with the heart failure (HF) develops cardiac tamponade. After the healthcare provider performs pericardiocentesis, which intervention is most important for the nurse to implement?
Monitor for recurrence of tamponade and cardiovascular collapse.

A client is admitted with an exacerbation of heart failure secondary to COPD. Which observation by the nurse require immediate intervention to reduce the likelihood of harm to this client? (Select all that apply)

  • A full pitcher of water is on the bedside table.
  • The client is lying in a supine position in bed.

A client’s telemetry monitor indicates ventricular fibrillation (VF). Which action should the nurse implement immediately?
Defibrillate with one shock.

The nurse is providing teaching to a client about management of type 2 diabetes mellitus. Which information stated by the client indicates understanding?
Avoid seasoning foods with salt and salt-containing spices

A client is being treated for syndrome of inappropriate antidiuretic hormone (SIADH). On examination, the client has a weight gain of 4.4 lbs (2kg) in 24 hours and an elevated blood pressure. Which intervention should the nurse implement first?
Ensure client takes a diuretic every morning

In providing nursing care for a client after gastric endoscopy, which commonly occurring problem should the nurse include interventions for in the post-procedure plan of care?
Sore throat.

While caring for a client who had an exploratory laparotomy yesterday, the nurse notes that coffee ground material is draining from the nasogastric tube. Which intervention should the nurse implement?
Test nasogastric drainage for the presence of blood.

A client who is HIV positive receives a prescription for megestrol 400 mg daily. Which finding should the nurse identify as a therapeutic response to this prescription?
Increased appetite.

What is the primary purpose the initiating nursing intervention that promote good nutrition, rest and exercise, and stress reduction for clients diagnosed with an HIV infection?
Improve function of the immune system.

A client with megaloblastic anemia who takes folate 1 mg by mouth daily reports to the nurse of feeling increasingly fatigued. Which laboratory value should the nurse review?
Complete blood count.

The nurse is providing discharge instructions to a client with atopic dermatitis who received a prescription for dexamethasone 8 mg by mouth daily. Which symptom should the nurse explain is most important for the client to report to the healthcare provider?
Rapid weight gain.

What is the primary goal when planning nursing care for a client with degenerate joint disease (DJD)?
Achieve satisfactory pain control.

The nurse is preparing a client for discharge who was hospitalized with a flare of symptoms from rheumatoid arthritis. Which instruction is most important for the nurse to include?
Take prescribed cortisone accurately

An adult male who fell from a roof and fractured his left femur is admitted for surgical stabilization after having a soft cast applied in the emergency department. Which assessment finding warrants immediate intervention by the nurse?
Onset of mild confusion.

The nurse is assessing a client who recently had an upper respiratory infection and now presents to the emergency department with lower extremity numbness and difficulty swallowing. Based on these findings, this client is at greater risk for which pathophysiological condition?
Guillain-Barre syndrome

Several months after a foot injury, an adult woman is diagnosed with neuropathic pain. The client describes the pain as severe and burning and is unable to put weight on her foot. She asks the nurse when the pain will “finally go away.” How should the nurse respond?
Assist the client in developing a goal of managing the pain.

A client with metastatic bone cancer is requesting pain medication. Which approach should the nurse use to assess the quality of the client’s pain?
Ask the client to describe the pain

A client receiving radiation therapy for squamous papilloma of the hard palate is experiencing mouth pain. Which is the best initial nursing action?
Administer a topical analgesic.

A client with cancer develops tumor Luis syndrome (TLS) following chemotherapy. Which nursing action has the highest priority in responding to the symptoms of this syndrome?
Maintain intravenous therapy.

The nurse on the medical-surgical unit is receiving a transfer report from the post-anesthesia care unit (PACU) nurse for a client who had an exploratory laparotomy. The PACU nurse provides the following information: “1000 mL normal saline is infusing at 125 mL/hr into the left wrist with 600 mL remaining. Ondansetron 4 mg intravenously every 8 hours is prescribed for nausea. The last dose was administered at 0700. The client is currently describing pain at a level 2 on a 0 to 10 pain scale. The client has a prescription for hydromorphone 1 mg intravenously every 2 hours as needed for pain. The last dose was administered at 1000.” Which additional information should the PACU nurse report?
Soft abdomen, absent bowel sounds, no bleeding on dressing.

The nurse places a client in the supine position on the operating table for abdominal surgery that is expected to extend beyond 2 hours. Which measure is most important for the nurse to provide?
Place protective padding at all dorsal pressure points.

While caring for a client’s postoperative dressing, the nurse observes a red and swollen wound with a moderate amount of yellow and green drainage and a foul odor. Before reporting this finding to the healthcare provider, the nurse should note which of the client’s laboratory values?
Neutrophil count.

A client with purulent drainage from an abdominal surgical incision is admitted with a possible vancomycin-resistant enterococci (VRE) infection. Which nursing interventions should the nurse include in the plan of care? (Select all that apply.)
Monitor the client’s white blood cell count.
Institute contact precautions for staff and visitors.
Send wound drainage for culture and sensitivity.

A client with ureterolithiasis is preparing for discharge after a ureteroscopic removal. Which instruction should the nurse include in the client’s postoperative discharge teaching?
Monitor urinary stream for decreased output.

An 18-year-old female client is seen at the health department for treatment of condylomata acuminata (perineal warts) caused by the human Papillomavirus (HPV). Which intervention should the nurse implement?
Reinforce the importance of annual Papanicolaou (pap) smears.

A female client calls the clinic with a report of fever, dizziness, and a diffuse rash. When obtaining a health history, the nurse confirms the use of tampons for her menstrual cycle. Which instruction should the nurse provide?
Seek immediate emergency medical care.

While transferring a client with a chest tube from the bed to a stretcher, the chest tube becomes disconnect from the water-seal drainage container. The nurse immediately immerses the end of the tube in a container of sterile water. Which action should the nurse implement next?
Prepare a new water-seal system and reattach the chest tube.

The nurse is caring for a client with a chronic obstructive pulmonary disease (COPD) who uses oxygen at 2 L/min per nasal cannula continuously. The nurse observes that the client is having increased shortness of breath with respirations at 23 breaths/minute. Which action should the nurse implement first?
Auscultate the client’s bilateral lung sounds and oxygen saturation.

During orientation, a newly hired nurse demonstrates suctioning of a tracheostomy in a skills class, as seen on the video. After the demonstration, the supervising nurse expresses concern that the demonstrated procedure increased the client’s risk for which problem?
Impaired gas exchange

The nurse is caring for a client who reports sudden right-sided numbness and weakness of the arm and leg. The nurse also observes a distinct right-sided facial droop. After reporting the findings to the healthcare provider, the nurse receives several prescriptions for the client, including a STAT computerized tomography scan of the head. Which intervention should the nurse perform in the immediate management of the client?
Obtain a focused history to determine recent bleeding and use of anticoagulants

An older client with heart failure (HF), coronary artery (CAD), and hypertension (HTN), is receiving these daily prescriptions: atenolol, furosemide, and enalapril. Which assessments should the nurse include in evaluating the effectiveness of the medications? (Select all that apply)

  • Blood pressure.
  • Daily weight

A client who is admitted with an acute coronary syndrome (ACS) receives eptifibatide, a glycoprotein (GP) IIB IIIA inhibitor. Which assessment finding places the client at greatest risk?
Unresponsive to painful stimuli.

The nurse is preparing to administered an IV dose of ciprofloxacin to a client with a urinary tract infection. Which client data requires the most immediate intervention by the nurse?
Serum creatinine of 4.5 mg/dL (398 mcmol/L).

An older client is admitted with pneumonia, and the healthcare provider prescribes penicillin G potassium intravenously. Which assessment finding increases the risk of adverse reactions in this client?
Daily use of spironolactone for hypertension.

Which part of the vascular system is affected most by blood pressure from minute to minute?
Capillary refill

The nurse is performing a routine assessment of an IV site for a client receiving both IV fluids and medications through the line. The client reports tenderness when the nurse touches the arm above the site. Which finding should the nurse expect which will require immediate intervention?
Red streak tracking the vein.

A male client with heart failure becomes short of breath, anxious, and has audible wheezing with pink frothy sputum. The nurse sits the client upright and provides oxygen per nasal cannula. The nurse receives a prescription to administered a one- time dose of morphine sulfate intravenously. What action should the nurse take?
Administered the dose of morphine sulfate as prescribed.

A client with pancreatitis complains of severe epigastric pain, so the nurse administers a prescribed narcotic analgesic. Ten minutes later, the client insists on sitting up and leaning forward. Which intervention should the nurse implement?
Position bedside table so the client can lean across it.

The nurse should expect a client diagnosed with regional enteritis (Crohn’s disease) ot exhibit what initial symptoms?
diarrhea, abdominal pain, and weight loss

The nurse is caring for a client with irritable bowel syndrome who presents with diarrhea and cramping abdominal pain. Which pathophysiologic mechanism supports this client’s clinical presentation?
tissue swelling die to an autoimmune response

The nurse is managing the care of a client who has partial-thickness burns over 60% of the body. The client’s risk for acquiring bacterial infections is increased due to which factors?
epithelium produces antimicrobial peptides that kill bacteria
acidic nature of skin protects against bacterial invasion

The nurse receives report on four clients who are complaining of increased pain. Which client requires immediate intervention by the nurse?
sharp pain related to a crushed femur

The nurse is performing an admission assessment of a client with generalized malaise and non-specific symptoms of not feeling well. Which finding is most important for the nurse report to the healthcare provider?
the trachea is to the right of the suprasternal notch

A male client with suspected lung cancer tells the nurse that he does not want to have a schedules biopsy performed, and states, “They already found the cancer when I had my chest x-ray.” Which action should the nurse implement?
instruct the client that biopsy results are important to determine the best treatment

A middle-aged client, diagnosed with Graves’ disease, asks the nurse about this condition. Which etiological pathology should the nurse include in the teaching plan about hyperthyroidism?

  • T3 and T4 hormone levels are increased
  • Large protruding eyeballa are a sign of hyperthyroidism
  • Graves’ disease, an autoimmune condition, affects thyroid stimulating hormone receptors

A 6-month-old infant is admitted to the hospital with diarrhea. The parent is feeding the infant a bottle of tap water and tells the nurse that the baby has taken three 8-ounce bottles of water in the last 4 hours. Which laboratory finding is most important for the nurse to monitor?
Serum sodium levels

The nurse is developing the plan of care for a hospitalized child with von Willebrand’s disease. What priority nursing interventions should be included in this child’s plan of care?
Guard against bleeding injuries

The nurse assesses a child in a 90-90 skeletal traction. Where should the nurse assess for signs of compartment syndrome?
tip of the toes of the foot that is on traction

The mother of a child with cerebral palsy asks the nurse if her child’s impaired movements will worsen as the child grows. Which response provides the best explanation?
brain damage with CP is not progressive but does have a variable course

The nurse is caring for an adolescent who fell 20 feet 5 months ago while climbing the side of a cliff and has been in a sustained vegetative state since the accident. Which intervention should the nurse implement?
talk directly to the adolescent while providing care

A 4-year-old-child is brought to the emergency department by a parent after being bitten by a non-venomous snake. The child is anxious and fearful, with a heart rate of 120 bpm and respirations of 42 bpm. The nurse anticipates the child developing which acid base imbalance?
respiratory alkalosis

The school nurse is called to the soccer field because a child has a nosebleed. I which position should the nurse place the child?
Sitting up and leaning forward.

The nurse caring for a toddler with Kawasaki disease who is receiving intravenous immunoglobulin for severe coronary inflammation is planning discharge teaching for the parents. Which instruction should the nurse provide during the discharge education?
plan for a follow-up echocardiogram occurring in 4-6 weeks

The mother of a 10-year-old who has sickle cell anemia expresses concern that her child will become addicted to pain medication if she follows the instructions on the prescription. Which information is most important for the nurse to provide?
giving pain medication around the clock will help control the pain

After receiving a change of shift report for clients on a medical surgical unit, which activity should the nurse assign to unlicensed assistive personnel?
transport a client with an IV to the radiology department

A client with Neisseria meningitidis calls the nurses station to report a severe headache and vomiting. The unlicensed assistive personnel approaches the room to provide an emesis basin and is stopped by the nurse. Which action should the nurse take?
Review the need for the UAP to wear a face mask while in close contact with the client

Nurses working on a surgical unit are concerned about a physician’s treatment of clients during invasive procedures, such as dressing changing and insertion of IV lines. Clients are often crying during the procedures, and the physician is usually unconcerned or annoyed by the client’s response. To resolve this problem, which actions should the nurse take?

  1. talk to the physician as a group in a non-confrontational manner
  2. document concerns and report them to the charge nurse
  3. submit a written report to the director of nursing
  4. contact the hospital’s Chief of Medical Services
  5. File a formal complaint with the state medical board

The health care provider prescribes a placebo instead of pain medication. What intervention should the nurse implement.

  • discuss ethical concerns about placebo use with the healthcare provider

The nurse enters a client’s room to administer oral medications and finds unlicensed assistive personnel providing person the client, whose condition has obviously deteriorated. The client is lying in a supine position and is weak, pale, and diaphoretic. Which priority nursing action is taken?
Advise the UAP to stop providing care so the nurse can assess the client’s condition

The nurse provides teaching about a schedule procedure to a male client who was admitted for diagnostic testing to determine the extent of metastasis of his cancer. An hour later the client asks the nurse for information about the scheduled procedure. What action should the nurse implement?

  • Repeat the client teaching and leave written instructions for the client

The nurse is preparing to send a client to the cardiac catheterization lab for elective cardioversion. Which intervention should the nurse implement before the client leaves the medical unit?
Document that the client has remained NPO

The nurse is auscultating a client’s heart sounds. Which description should the nurse use to document this sound?
S1 S2 S3

The nurse observes the practical nurse placing a client on the right side with the right arm behind the head for a scheduled echocardiogram. Which action should the nurse implement?
demonstrate to the PN how to position the client more effectively for the procedure

A female client with fibromyalgia asks the nurse to arrange for hospice care to help her manage the severe, chronic pain. Which intervention should the nurse provide to address the client’s problem?
arrange an appointment with a pain specialist

An adult male with end-stage liver disease has been unresponsive for the past 3 days. This electroencephalography reveals no active brain activity. The family wants to discontinue feeding and donate his viable organs. Which action should the nurse to take?

  • Contact the regional organ procurement agency

A client on the cardiac telemetry unit unexpectedly begin manifesting ventricular fibrillation and the advanced cardiac life support team defibrillates the client, restoring a normal sinus rhythm. Later in the day, a family member questions why the code was called, telling the nurse that the client has a living will. How should the nurse respond.
Seek clarification of the type of advance directive the client has.

Four days after exposure to the coronavirus a client has a negative COVID-19 test result. Eight days after the negative test result, the client presents with fever, fatigue, and cough and the nurse performs a second COVID-19 test. Which action is most important for the nurse to take?
move the client to a private room, keep the door closed, and initiate droplet precaution

A client with a gram-negative bacterial infection develops disseminated intravascular coagulation. Which intervention should the nurse prioritize in the client’s plan of care?
Maintain vascular volume with fluid replacement

A client with an active gastrointestinal bleeding ulcer is to receive two units packed red blood cells. By rapid infusion. Vital signs are 96F, heart rate 132 beats/minute, respirations 22 bpm, and blood pressure 100/80 mmhg, And peripheral oxygen saturation 85%. Based on these findings, which intervention should the nurse implement first?
insert two large bore intravenous catheters

The nurse is caring for a client admitting to the unit for testing of suspected hyperthyroidism. The client describes weakness, nervousness, a racing heart beat, and recent weight loss of 15 pounds. Which action should the nurse implement first?
Administer beta-adrenergic blocking agent

The nurse is preparing an adult with Addison’s disease for self-management. Which information should the nurse include in the client’s instructions?
Events requiring steroid dose adjustments

A client who is obese reports severe pain and is unable to bear weight in the right ankle after making dietary changes 3 weeks ago for weight loss. The client’s medical history includes hypertension, gouty arthritis, and cholecystitis. Which instruction should the nurse include in the discharge teaching?
Avoid the consumption of wine, beer, and coffee

A male client with a brain tumor is scheduled for a biopsy in the morning. During the admission procedure, the client has a tonic-clonic seizure that lasts 50 sec. Following the seizure, the client is lethargic and confused and his wife tells the nurse that her husband has never had a seizure before and has always been alert and communicative. Which action should the nurse take?
Explain the postictal state that usually follows seizure

The nurse prepares to teach four clients about diabetes mellitus. Which client has the greatest need for instructions?
An adult who has type 2 diabetes and has a glycosylated hemoglobin at 10%

A client who is newly diagnosed with type 2 diabetes mellitus receives a prescription for metformin 500mg PO twice daily. What information should the nurse include in this clients teaching plan?

  • Report persistent polyuria to the healthcare provider
  • Take metformin with the morning and evening meal
  • Recognize signs and symptoms of hypoglycemia

An older adult client is diagnosed with severe shingles and starts a new prescription for acyclovir, an antiviral medication. Which action should the nurse include during client teaching prior to discharge?
Encourage increased oral fluid intake while taking the medication

The nurse is providing education to a client who receives a prescription for zolpidem. Which information about the medication should the nurse include?
Take before bedtime

A client who is a veteran comes to the emergency department appearing tense, anxious, and having difficulty concentrating on the questions the nurse is asking
during the health history. Which client statement is most important for the nurse to document?
“I can’t forgive myself for leaving my buddy behind”

The nurse notes that a client with depression has been more withdrawn and non-communicative during the past two weeks. Which intervention is most important to include in the updated plan of care for this client?
Engage the client in non-threatening conversations

A male client who arrives in the Emergency Department after a motor vehicle collision tells the nurse, “The care started to slide, and I just decided to let it go. Everyone would be better off if I was no longer around.” How should the nurse respond?
Ask the client of the MVC was a suicide attempt

The nurse is planning care for a client who admits having suicidal thoughts. Which client behavior indicates the highest risk for the client acting on these suicidal thoughts?
Begins to show signs of improvement in affect

The nurse notices that a male client is particularly delusional one afternoon. He begins to pace the floor and appears to be losing control of himself. Which intervention is best for the nurse to implement?
Move the client to a quiet place on the unit

A mother brings her 2-month-old infant to the clinic for a well-baby appointment. The nurse obtains a history and conducts a physical assessment. Which finding requires the most immediate intervention?
Bilateral retinal hemorrhages

A client diagnosed with schizophrenia is prescribed the atypical antipsychotic clozapine. Which intervention should the nurse include in the discharge teaching?
Discuss the importance of checking the white blood count weekly

An older client with dementia who is refusing to allow an unlicensed assistive personnel bathe he, is becoming increasingly agitated and stating the UAP want to hurt her and tie her up. Which approach should the nurse use with the client?
Use distraction and therapeutic communication skills

A woman with an anxiety disorder called her obstetrician’s office and tells the nirse of increased anxiety since the normal vaginal delivery of her son three weeks ago. Since she is breastfeeding, she stopped taking her antianxiety medication, but thinks she may need to start taking them again because of her increased anxiety. What response is best for the nurse to provide this woman?
Inform her that soe antianxiety medication are safe to take while breastfeeding

The nurse is interacting with a client who is diagnosed with postpartum depression. Which finding should the nurse document as objective signs of depression?

  • interacts with a flat affect
  • Has a disheveled appearance

AN adult client who is admitted to the mental health unit for treatment of bipolar disorder has slightly slurred speech pattern and an unsteady gait. Which assessment finding is most important for the nurse to report to the healthcare provider?
Serum lithium of 1.6 mEq/L or mmol/L

The nurse is planning group therapy for clients in a substance abuse program. The focus of the group is, “Risk factors for hepatitis.” What intervention should the nurse plan for the gorup?
Summarize what the group talked about

At 1130, the nurse assumes care of an adult client with diabetes mellitus who was admitted with an infected foot ulcer. After reviewing the client’s electronic heath record, which priority nursing action should the nurse implement?
Administer insulin per sliding scale

The nurse observes unlicensed assistive personnel begin to provide oral care to an unresponsive client who is at risk for aspiration as seen in the picture. Which instruction should the nurse provide the UAP?

  • Turn the clients head to the side
  • Remove the gloved finger from the mouth
  • elevate the head of bed to semi-fowler’s

Suicide precautions are initiated for a child admitted to the mental health unit following an intentional narcotic overdose. After a visitor leaves, the nurse finds a package of cigarettes in the client’s room. Which intervention is most important for the nurse to implement?
Remove cigarettes for the client’s room

A family member of a frail elderly adult asks the nurse about eligibility requirements for hospice care. What information should the nurse provide? (Select all that apply.)

A.)A client must be willing to accept palliative care, not curative care.

B.)The healthcare provider must project that the client has 6 months or less to live.

C.)The client must be diagnosed with clinical depression

D.)The client must be of sound mind
A,B

A client with atrial fibrillation receives a new prescription for dabigatran. What instruction should the nurse include in this client’s teaching plan?
Avoid use of nonsteroidal ant-inflammatory drugs (NSAID).

An infant who is admitted for surgical repair of a ventricular septal defect (VSD) is irritable and diaphoretic with jugular vein distention. Which prescription should the nurse administer first?
Digoxin

The nursing staff on a medical unit includes a registered nurse (RN), practical nurse (PN), and an unlicensed assistive personnel (UAP). Which task should the charge nurse assign to the RN?
Supervise a newly hired graduate nurse during an admission assessment.

While teaching a young male adult to use an inhaler for his newly diagnosed asthma, the client stares into the distance and appears to be concentrating on something other than the lesson the nurse is presenting. What action should the nurse take?
Ask the client what he is thinking about at his time.

After several hours of non-productive coughing, a client presents to the emergency room complaining of chest tightness and shortness of breath. History includes end stage chronic obstructive pulmonary disease (COPD) and diabetes mellitus. While completing the pulmonary assessment, the nurse hears wheezing and poor air movement bilaterally. Which actions should the nurse implement? (Select all that apply.)

A.)Administer PRN nebulizer treatment.

B.)Obtain 12 lead electrocardiogram.

C.)Monitor continuous oxygen saturation.

D.) Lay the client in the prone position
A,B,C

The nurse caring for a 3-month-old boy one day after a pylorotomy notices that the infant is restless, is exhibiting facial grimaces, and is drawing his knees to his chest. What action should the nurse take?
Administer a prescribed analgesia for pain.

A 4-year-old with acute lymphocytic leukemia (ALL) is receiving a chemotherapy (CT) protocol that includes methotrexate (Mexate, Trexal, MIX), an antimetabolite. Which information should the nurse provide the parents about caring for their child?
Use sunblock or protective clothing when outdoors.

Two days after admission a male client remembers that he is allergic to eggs, and informs the nurse of the allergy. Which actions should the nurse implement? SATA

A.) Tell the client that its a mild reaction

B.)Notify the food services department of the allergy.

C.)Enter the allergy information in the client’s record.

D.)Add egg allergy to the client’s allergy arm band.
B,C,D

The rapid response team’s detects return of spontaneous circulation (ROSC) after 2 min of continuous chest compressions. The client has a weak, fast pulse and no respiratory effort, so the healthcare provider performs a successful oral, intubation. What action should the nurse implement?
Perform bilateral chest auscultation.

After administering an antipyretic medication. Which intervention should the nurse implement?
Encouraging liberal fluid intake

A client with hyperthyroidism is being treated with radioactive iodine (I-131). Which explanation should be included in preparing this client for this treatment?
Describe radioactive iodine as a tasteless, colorless medication administered by the healthcare provider

After a colon resection for colon cancer, a male client is moaning while being transferred to the Postanesthesia Care Unit (PACU). Which intervention should the nurse implement first?
Determine client’s pulse, blood pressure, and respirations

The nurse is caring for a group of clients with the help of a licensed practical nurse (LPN) and an experienced unlicensed assistive personnel (UAP). Which procedures can the nurse delegate to the UAP? (Select all that apply)

A.)Take postoperative vital signs for a client who has an epidual following knee arthroplasty

B.)Collect a sputum specimen for a client with a fever of unknown origin

C.)Ambulate a client who had a femoral-popliteal bypass graft yesterday
A,B,C

A male client with cirrhosis has ascites and reports feeling short of breath. The client is in semi Fowler position with his arms at his side. What action should the nurse implement?
Raise the head of the bed to a Fowler’s position and support his arms with a pillow

A client with a history of chronic pain requests a nonopioid analgesic. The client is alert but has difficulty describing the exact nature and location of the pain to the nurse. Which action should the nurse implement next?
Administer the analgesic as requested

The nurse uses the parkland formula (4ml x kg x total body surface area = 24 hours fluid replacement) to calculate the 24-hours IV fluid replacement for a client with 40% burns who weighs 76kg. How many ml should the client receive? (Enter numeric value only.)
12160

A client with leukemia undergoes a bone marrow biopsy. The client’s laboratory values indicate the client has thrombocytopenia. Based on this data, which nursing assessment is most important following the procedure?

A.)Observe aspiration site.

B.)Assess body temperature

C.)Monitor skin elasticity

D.)Measure urinary output
A

An 18-year-old female client is seen at the health department for treatment of condylomata acuminate (perineal warts) caused by the human papillomavirus (HPV). Which intervention should the nurse implement?
Reinforce the importance of annual papanicolaou (Pap) smears.

A client admitted to the psychiatric unit diagnosed with major depression wants to sleep during the day, refuses to take a bath, and refuses to eat. Which nursing intervention should the nurse implement first?
Establish a structured routine for the client to follow.

A client with history of bilateral adrenalectomy is admitted with a week, irregular pulse, and hypotension. Which assessment finding warrants immediate intervention by the nurse?
Ventricular arrhythmias.

The mother of a 7-month-old brings the infant to the clinic because the skin in the diaper area is excoriated and red, but there are no blisters or bleeding. The mother reports no evidence of watery stools. Which nursing intervention should the nurse implement?
Instruct the mother to change the child’s diaper more often.

A resident of a long-term care facility, who has moderate dementia, is having difficulty eating in the dining room. The client becomes frustrated when dropping utensils on the floor and then refuses to eat. What action should the nurse implement?
Encourage the client to eat finger foods.

A client is receiving mesalamine 800 mg PO TID. Which assessment is most important for the nurse to perform to assess the effectiveness of the medication?
Bowel patterns

While in the medical records department, the nurse observes several old medical records with names visible in waste container. What action should the nurse implement?
Contact the medical records department supervisor.

A 16-year-old adolescent with meningococcal meningitis is receiving a continuous IV infusion of penicillin G, which is prescribed as 20 million units in a total volume of 2 liters of normal saline every 24 hr. The pharmacy delivers 10 million units/ liters of normal saline. How many ml/hr should the nurse program the infusion pump? (Enter numeric value only. If rounding is required, round to the nearest whole number.)
83

While visiting a female client who has heart failure (HF) and osteoarthritis, the home health nurse determines that the client is having more difficulty getting in and out of the bed than she did previously. Which action should the nurse implement first?
Submit a referral for an evaluation by a physical therapist.

A client has an intravenous fluid infusing in the right forearm. To determine the client’s distal pulse rate most accurately, which action should the nurse implement?
Submit a referral for an evaluation by a physical therapist.

A child is admitted to the pediatric unit diagnosed with sickle cell crisis. When the nurse walks into the room, the unlicensed assistive personnel (UAP) is encouraging the child to stay in bed in the supine position. Which action should the nurse implement?
Reposition the client with the head of the bed elevated.

  1. After six days on a mechanical ventilator, a male client is extubated and place on 40% oxygen via face mask. He is awake and cooperative, but complaining of a severe sore throat. While sipping water to swallow a medication, the client begins coughing, as if strangled. What intervention is most important for the nurse to implement?
    Hold oral intake until swallow evaluation is done.

The nurse is interacting with a female client who is diagnosed with postpartum depression. Which finding should the nurse document as an objective signs of depression? (Select all that apply)

A.)Interacts with a flat affect.

B.)Avoids eye contact.

C.)Makes dull eye contact

D.)Has a disheveled appearance.
A,B,D

A client in the postanesthesia care unit (PACU) has an eight (8) on the Aldrete postanesthesia scoring system. What intervention should nurse implement?
Transfer the client to the surgical floor.

In caring for the body of a client who just died, which tasks can be delegate to the unlicensed assistive personnel (UAP)? (Select all that apply.)

A.)Place personal religious artifacts on the body.

B.)Attach identifying name tags to the body.

C.)Follow cultural beliefs in preparing the body.

D.) Inform the family
A,B,C

An adult male reports the last time he received penicillin he developed a severe maculopapular rash all over his chest. What information should the nurse provide the client about future antibiotic prescriptions?
Be alert for possible cross-sensitivity to cephalosporin agents.

A client with a prescription for “do not resuscitate” (DNR) begins to manifest signs of impending death. After notifying the family of the client’s status, what priority action should the nurse implement?
The client’s need for pain medication should be determined.

A client with cirrhosis of the liver is admitted with complications related to end stage liver disease. Which intervention should the nurse implement? (Select all that apply.)

A.)Monitor abdominal girth.

B.)Increase oral fluid intake to 1500 ml daily.

C.)Report serum albumin and globulin levels.

D.)Provide diet low in phosphorous.

E.)Note signs of swelling and edema.
A,C,E

During discharge teaching, the nurse discusses the parameters for weight monitoring with a client who was recently diagnosed with heart failure (HF). Which information is most important for the client to acknowledge?
Report weight gain of 2 pounds (0.9kg) in 24 hours

Which problem, noted in the client’s history, is important for the nurse to be aware of prior to administration of a newly prescribed selective serotonin reuptake inhibitor (SSRI)?
Aural migraine headaches.

When implementing a disaster intervention plan, which intervention should the nurse implement first?

A.)Initiate the discharge of stable clients from hospital units

B.)Identify a command center where activities are coordinated

C.)Assess community safety needs impacted by the disaster

D.)Instruct all essential off-duty personnel to report to the facility
B

The nurse is evaluating a client’s symptoms, and formulates the nursing diagnosis, “high risk for injury due to possible urinary tract infection.” Which symptoms indicate the need for this diagnosis?
Fever and dysuria.

A client is admitted with metastatic carcinoma of the liver, ascites, and bilateral 4+ pitting edema of both lower extremities. When the client complains that the antiembolic stocking are too constricting, which intervention should the nurse implement?
Maintain both lower extremities elevated on pillows.

A client with muscular dystrophy is concerned about becoming totally dependent and is reluctant to call the nurse to assist with activities of daily living (ADLs). To achieve maximum mobility and independence, which intervention is most important for the nurse to include in the client’s plan of care?
Teach family proper range of motion exercises.

The nurse is teaching a postmenopausal client about osteoporosis prevention. The client reports that she smokes 2 packs of cigarettes a day and takes 750 mg calcium supplements daily. What information should the nurse include when teaching this client about osteoporosis prevention?
Postmenopausal women need an intake of at least 1,500 mg of calcium daily.

When evaluating a client’s rectal bleeding, which findings should the nurse document?
Color characteristics of each stool.

The nurse is auscultating a client’s lung sounds. Which description should the nurse use to document this sound?

A.)High pitched or fine crackles.

B.)Rhonchi

C.)High pitched wheeze

D.)Stridor
A

An adult male is admitted to the emergency department after falling from a ladder. While waiting to have a computed tomography (CT) scan, he requests something for a severe headache. When the nurse offers him a prescribed does of acetaminophen, he asks for something stronger. Which intervention should the nurse implement?
Explain the reason for using only non-narcotics.

The nurse is managing the care of a client with Cushing’s syndrome. Which interventions should the nurse delegate to the unlicensed assistive personnel (UAP)? (Select all that apply)
Weigh the client and report any weight gain.

Report any client complaint of pain or discomfort.

Note and report the client’s food and liquid intake during meals and snacks.

Ten years after a female client was diagnosed with multiple sclerosis (MS), she is admitted to a community palliative care unit. Which intervention is most important for the nurse to include in the client’s plan of care?
Medicate as needed for pain and anxiety.

An increased number of elderly persons are electing to undergo a new surgical procedure which cures glaucoma. What effect is the nurse likely to note as a result of this increases in glaucoma surgeries?
Decrease prevalence of glaucoma in the population.

The nurse is caring for a client who is entering the second stage of labor. Which action should the nurse implement first?
Convey to the client that birth is imminent.

To evaluate the effectiveness of male client’s new prescription for ezetimibe, which action should the clinic nurse implement?
Remind the client to keep his appointments to have his cholesterol level checked.

Diagnostic studies indicate that the elderly client has decreased bone density. In providing client teaching, which area of instruction is most important for the nurse to include?
Fall prevention measures.

A young adult client is admitted to the emergency room following a motor vehicle collision. The client’s head hit the dashboard. Admission assessment include: Blood pressure 85/45 mm Hg, temperature 98.6 F, pulse 124 beat/minute and respirations 22 breath/minute. Based on these data, the nurse formulates the first portion of nursing diagnosis as ” Risk of injury” What term best expresses the “related to” portion of nursing diagnosis?

A.)Infection

B.)Increase intracranial pressure

C.)Shock

D.)Head Injury.
C

An older male client with history of diabetes mellitus, chronic gout, and osteoarthritis comes to the clinic with a bag of medication bottles. Which intervention should the nurse implement first?
Identify pills in the bag.

A male client who was diagnosed with viral hepatitis A 4 weeks ago returns to the clinic complaining of weakness and fatigue. Which finding is most important for the nurse to report to the healthcare provider?
New onset of purple skin lesions.

In assessing a client twelve hours following transurethral resection of the prostate (TURP), the nurse observes that the urinary drainage tubing contains a large amount of clear pale pink urine and the continuous bladder irrigation is infusing slowly. What action should the nurse implement?
Ensure that no dependent loops are present in the tubing.

The healthcare provider prescribes the antibiotic Cefdinir (cephalosporin) 300mg PO every 12 h for a client with postoperative wound infections. Which feeds should the nurse encourage this client to eat?

A.)Yogurt and/or buttermilk.

B.)Avocados and cheese

C.)Green leafy vegetables

D.)Fresh fruits
A

The charge nurse is making assignment on a psychiatric unit for a practical nurse (PN) and newly license register nurse (RN). Which client should be assigned to the RN?

A.)An adult female who has been depress for the past several month and denies suicidal ideations.

B.)A middle-age male who is in depressive phase on bipolar disease and is receiving Lithium.
C.)A young male with schizophrenia who said voices is telling him to kill his psychiatric.

D.)An elderly male who tell the staff and other client that he is superman and he can fly.
C

In assessing an older female client with complication associated with chronic obstructive pulmonary disease (COPD), the nurse notices a change in the client’s appearance. Her face appears tense and she begs the nurse not to leave her alone. Her pulse rate is 100, and respirations are 26 per min. What is the primary nursing diagnosis?
Anxiety related to fear of suffocation.

A client with a cervical spinal cord injury (SCI) has Crutchfield tongs and skeletal traction applied as a method of closed reduction. Which intervention is most important for the nurse to include in the client’s a plan of care?
Provide daily care of tong insertion sites using saline and antibiotic ointment

A client arrives on the surgical floor after major abdominal surgery. What intervention should the nurse perform first?
Determine the client’s vital sign.

A client is admitted to the emergency department with a respiratory rate of 34 breaths per minute and high pitched wheezing on inspiration and expiration, the medical diagnosis is severe exacerbation of asthma. Which assessment finding, obtained 10 min after the admission assessment, should the nurse report immediately to the emergency department healthcare provider?
No wheezing upon auscultation of the chest.

The nurse is planning a class for a group of clients with diabetes mellitus about blood glucose monitoring. In teaching the class as a whole, the nurse should emphasize the need to check glucose levels in which situation?
During acute illness

A 350-bed acute care hospital declares an internal disaster because the emergency generators malfunctioned during a city-wide power failure. The UAPs working on a general medical unit ask the charge nurse what they should do first. What instruction should the charge nurse provide to these UAPs?
Tell all their assigned clients to stay in their rooms.

The healthcare provider changes a client’s medication prescription from IV to PO administration and double the dose. The nurse notes in the drug guide that the prescribed medication, when given orally, has a high first-pass effect and reduce bioavailability. What action should the nurse implement?
Administer the medication via the oral route as prescribed

A client refuses to ambulate, reporting abdominal discomfort and bloating caused by “too much gas buildup” the client’s abdomen is distended. Which prescribed PRN medication should the nurse administer?
Simethicone (Mylicon)

The public nurse health received funding to initiate primary prevention program in the community. Which program the best fits the nurse’s proposal?

A.)Lead screening for children in low-income housing.

B.)Case management and screening for clients with HIV

C.)Regional relocation center for earthquake victims

D.)Vitamin supplements for high-risk pregnant women.
D

When assessing and adult male who presents as the community health clinic with a history of hypertension, the nurse note that he has 2+ pitting edema in both ankles. He also has a history of gastroesophageal reflex disease (GERD) and depression. Which intervention is the most important for the nurse to implement?

A.)Arrange to transport the client to the hospital

B.)Instruct the client to keep a food journal, including portions size.

C.)Review the client’s use of over the counter (OTC) medications.

D.)Reinforce the importance of keeping the feet elevated.
C

An older client is admitted to the intensive care unit with severe abdominal pain, abdominal distention, and absent bowel sound. The client has a history of smoking 2 packs of cigarettes daily for 50 years and is currently restless and confused. Vital signs are: temperature 96`F, heart rate 122 beats/minute, respiratory rate 36 breaths/minute, mean arterial pressure(MAP) 64 mmHg and central venous pressure (CVP) 7 mmHg. Serum laboratory findings include: hemoglobin 6.5 grams/dl, platelets 6o, 000, and white blood cell count (WBC) 3,000/mm3. Based on these findings this client is at greatest risk for which pathophysiological condition?

A.)Multiple organ dysfunction syndrome (MODS)

B.)Disseminated intravascular coagulation (DIC)

C.)Chronic obstructive disease.

D.)Acquired immunodeficiency syndrome (AIDS)
A

A man expresses concern to the nurse about the care his mother is receiving while hospitalized. He believes that her care is not based on any ethical standards and ask what type of care he should expect from a public hospital. What action should the nurse take?
Provide the man and his mother with a copy of the Patient’s Bill of Rights

A client experiencing withdrawal from the benzodiazepines alprazolam (Xanax) is demonstrating severe agitation and tremors. What is the best initial nursing action?

A.)Administer naloxone (Narcan) per PNR protocol

B.)Initiate seizure precautions

C.)Obtain a serum drug screen

D.)Instruct the family about withdrawal symptoms.
B

The nurse is caring for a client who is taking a macrolide to treat a bacterial infection. Which finding should the nurse report to the healthcare provider before administering the next dose?

A.)Jaundice

B.)Nausea

C.)Fever

D.)Fatigue
A

A client with Alzheimer’s disease (AD) is receiving trazodone (Desyrel), a recently prescribed atypical antidepressant. The caregiver tells the home health nurse that the client’s mood and sleep patterns are improved, but there is no change in cognitive ability. How should the nurse respond to this information?

A.)Explain that it may take several weeks for the medication to be effective

B.)Confirm the desired effect of the medication has been achieved.

C.)Notify the health care provider than a change may be needed.

D.)Evaluate when and how the medication is being administered to the client.
B

A client with diabetic peripheral neuropathy has been taking pregabalin (Lyrica) for 4 days. Which finding indicates to the nurse that the medication is effective?

A.)Reduced level of pain

B.)Full volume of pedal pulses

C.)Granulating tissue in foot ulcer

D.)Improved visual acuity
A

A group of nurse-managers is asked to engage in a needs assessment for a piece of equipment that will be expensed to the organization’s budget. Which question is most important to consider when analyzing the cost-benefit for this piece of equipment?

A.)How many departments can use this equipment?

B.)Will the equipment require annual repair?
C.)Is the cost of the equipment reasonable?

D.)Can the equipment be updated each year?
A

While receiving a male postoperative client’s staples de nurse observe that the client’s eyes are closed and his face and hands are clenched. The client states, “I just hate having staples removed”. After acknowledgement the client’s anxiety, what action should the nurse implement?

A.)Encourage the client to continue verbalize his anxiety

B.)Attempt to distract the client with general conversation

C.)Explain the procedure in detail while removing the staples

D.)Reassure the client that this is a simple nursing procedure.
B

A male client is admitted for the removal of an internal fixation that was inserted for the fracture ankle. During the admission history, he tells the nurse he recently received vancomycin (vancomycin) for a methicillin-resistant Staphylococcus aureus (MRSA) wound infection. Which action should the nurse take? (Select all that apply.)

A.)Collect multiple site screening culture for MRSA

B.)Call healthcare provider for a prescription for linezolid (Zyrovix)

C.)Place the client on contact transmission precautions

D.)Obtain sputum specimen for culture and sensitivity

E.)Continue to monitor for client sign of infection.
A,C,E

A vacuum-assistive closure (VAC) device is being use to provide wound care for a client who has stage III pressure ulcer on a below-the- knee (BKA) residual limb. Which intervention should the nurse implement to ensure maximum effectiveness of the device?
Ensure the transparent dressing has no tears that might create vacuum leaks

The nurse is developing the plan of care for a client with pneumonia and includes the nursing diagnosis of “Ineffective airway clearance related to thick pulmonary secretions.” Which intervention is most important for the nurse to include in the client’s plan of care?
Increase fluid intake to 3,000 ml/daily

The nurse plans to collect a 24- hour urine specimen for a creatinine clearance test. Which instruction should the nurse provide to the adult male client?

A.)Clearance around the meatus, discard first portion of voiding, and collect the rest in a sterile bottle

B.)Urinate at specific time, discard the urine, and collect all subsequent urine during the next 24 hours.

C.)For the next 24 hours, notify the nurse when the bladder is full, and the nurse will collect catheterized specimens.

D.)Urinate immediately into a urinal, and the lab will collect specimen every 6 hours, for the next 24 hours.
D

The nurse is preparing to administer a histamine 2-receptor antagonist to a client with peptic ulcer disease. What is the primary purpose of this drug classification?
Decreases the amount of HCL secretion by the parietal cells in the stomach

The healthcare provider prescribes acarbose (Precose), an alpha-glucosidase inhibitor, for a client with Type 2 diabetes mellitus. Which information provides the best indicator of the drug’s effectiveness?
Hemoglobin A1C (HbA1C) reading less than 7%

The nurse assesses a client with new onset diarrhea. It is most important for the nurse to question the client about recent use of which type of medication?

A.)Antibiotics

B.)Anticoagulants

C.)Antihypertensive

D)Anticholinergics
A

A neonate with a congenital heart defect (CHD) is demonstrating symptoms of heart failure (HF). Which interventions should the nurse include in the infant’s plan of care?

A.)Give O2 at 6 L/nasal cannula for 3 repeated oximetry screens below 90%

B.)Administer diuretics via secondary infusion in the morning only

C.)Evaluate heart rate for effectiveness of cardio tonic medications

D.)Use high energy formula 30 calories/ounce at Q3 hours feeding via softnipples

E.)Ensure Interrupted and frequent rest periods between procedures.
A,C,D,E

The nurse is caring for a 4-year-old male child who becomes unresponsive as his heart rate decreases to 40 beats/minute. His blood pressure is 88/70 mmHg, and his oxygen saturation is 70% while receiving 100% oxygen by non-rebreather face mask. In what sequence, from first to last, should the nurse implement these actions? (Place the first action on top and last action on the bottom.)

Administer epinephrine 0.01 mg/kg intraosseous (IO)

Start chest compressions with assisted manual ventilations

Review the possible underlying causes for bradycardia

Apply pads and prepare for transthoracic pacing

  1. Start chest compressions with assisted manual ventilations
  2. Administer epinephrine 0.01 mg/kg intraosseous (IO)
  3. Apply pads and prepare for transthoracic pacing
  4. Review the possible underlying causes for bradycardia

An elderly male client is admitted to the mental health unit with a sudden onset of global disorientation and is continuously conversing with his mother, who died 50 years ago. The nurse reviews the multiple prescriptions he is currently taking and assesses his urine specimen, which is cloudy, dark yellow, and has foul odor. These findings suggest that his client is experiencing which condition?

A.)Psychotic episode

B.)Depression

C.)Dementia

D.)Delirium
D

A preschool-aged boy is admitted to the pediatric unit following successful resuscitation from a near-drowning incident. While providing care to child, the nurse begins talking with his preadolescent brother who rescued the child from the swimming pool and initiated resuscitation. The nurse notices the older boy becomes withdrawn when asked about what happened. What action should the nurse take?
Ask the older brother how he felt during the incident.

Following an esophagogastroduodenoscopy (EGD) a male client is drowsy and difficult to arouse, and his respiration are slow and shallow. Which action should the nurse implement? Select all that apply.

A.)Prepare medication reversal agent

B.)Check oxygen saturation level

C.)Apply oxygen via nasal cannula

D.)Initiate bag- valve mask ventilation.

E.)Begin cardiopulmonary resuscitation
A,B,C

The nurse is planning preoperative teaching plan of a 12-years old child who is scheduled for surgery. To help reduce the child anxiety, which action is the best for the nurse to implement?

A.)Give the child syringes or hospital mask to play it at home prior to hospitalization.

B.)Include the child in pay therapy with children who are hospitalized for similarsurgery.

C.)Provide a family tour of the preoperative unit one week before the surgery is scheduled.

D.)Provide doll an equipment to re-enact feeling associated with painful procedures.
C

Which intervention should the nurse implement during the administration of vesicant chemotherapeutic agent via an IV site in the client’s arm?
Assess IV site frequently for signs of extravasation

When development a teaching plan for a client newly diagnosed type 1 diabetes, the nurse should explain that an increase thirst is an early sing of diabetes ketoacidosis (DKA), which action should the nurse instruct the client to implement if this sign of DKA occur

A.)Resume normal physical activity

B.)Drink electrolyte fluid replacement

C.)Give a dose of regular insulin per sliding scale

D.)Measure urinary output over 24 hours.
C

The nurse is teaching a group of clients with rheumatoid arthritis about the need to modify daily activities. Which goal should the nurse emphasize?

A.)Protect joint function

B.)Improve circulation

C.)Control tremors

D.)Increase weight bearing
A

An adult client experiences a gasoline tank fire when riding a motorcycle and is admitted to the emergency department (ED) with full thickness burns to all surfaces of both lower extremities. What percentage of body surface area should the nurse document in the electronic medical record (EMR)?

A.)9 %

B.)18 %

C.)36 %

D.)45 %
C

A client with hyperthyroidism is receiving propranolol (Inderal). Which finding indicates that the medication is having the desired effect?

A.)Decrease in serum T4 levels

B.)Increase in blood pressure

C.)Decrease in pulse rate

D.)Goiter no longer palpable
C

An older male client with type 2 diabetes mellitus reports that has experiences legs pain when walking short distances, and that the pain is relieved by rest. Which client behavior indicates an understanding of healthcare teaching to promote more effective arterial circulation?

A.)Consistently applies TED hose before getting dressed in the morning.

B.)Frequently elevated legs thorough the day.

C.)Inspect the leg frequently for any irritation or skin breakdown

D.)Completely stop cigarette/ cigar smoking.
D

A community health nurse is concerned about the spread of communicable diseases among migrant farm workers in a rural community. What action should the nurse take to promote the success of a healthcare program designed to address this problem?
Establish trust with community leaders and respect cultural and family values

The nurse performs a prescribed neurological check at the beginning of the shift on a client who was admitted to the hospital with a subarachnoid brain attack (stroke). The client’s Glasgow Coma Scale (GCS) score is 9. What information is most important for the nurse to determine?

A.)The client’s previous GCS score

B.)When the client’s stroke symptoms started

C.)If the client is oriented to time

D.)The client’s blood pressure and respiration rate
A

The charge nurse in a critical care unit is reviewing clients’ conditions to determine who is stable enough to be transferred. Which client status report indicates readiness for transfer from the critical care unit to a medical unit?
Chronic liver failure with a hemoglobin of 10.1 and slight bilirubin elevation

Based on principles of asepsis, the nurse should consider which circumstance to be sterile?

A.)One inch- border around the edge of the sterile field set up in the operating room

B.)A wrapped unopened, sterile 4×4 gauze placed on a damp table top.

C.)An open sterile Foley catheter kit set up on a table at the nurse waist level

D.)Sterile syringe is placed on sterile area as the nurse riches over the sterile field.
C

An unlicensed assistive personnel (UAP) reports that a client’s right hand and fingers spasms when taking the blood pressure using the same arm. After confirming the presence of spams what action should the nurse take?

A.)Ask the UAP to take the blood pressure in the other arm

B.)Tell the UAP to use a different sphygmomanometer.

C.)Review the client’s serum calcium level

D.)Administer PRN antianxiety medication.
C

A 56-years-old man shares with the nurse that he is having difficulty making decision about terminating life support for his wife. What is the best initial action by the nurse?

A.)Provide an opportunity for him to clarify his values related to the decision

B.)Encourage him to share memories about his life with his wife and family

C.)Advise him to seek several opinions before making decision

D.)Offer to contact the hospital chaplain or social worker to offer support.
A

A client is being discharged home after being treated for heart failure (HF). What instruction should the nurse include in this client’s discharge teaching plan?

A.)Weigh every morning

B.)Eat a high protein diet

C.)Perform range of motion exercises

D.)Limit fluid intake to 1,500 ml daily
A

A woman just learned that she was infected with Heliobacter pylori. Based on this finding, which health promotion practice should the nurse suggest?
Encourage screening for a peptic ulcer

A client who recently underwear a tracheostomy is being prepared for discharge to home. Which instructions is most important for the nurse to include in the discharge plan?
Teach tracheal suctioning techniques

A child with heart failure is receiving the diuretic furosemide (Lasix) and has serum potassium level 3.0 mEq/L. Which assessment is most important for the nurse to obtain?

A.)Daily intake of foods rich in potassium.

B.)Cardiac rhythm and heart rate.

C.)Hourly urinary output

D.)Thirst ad skin turgor.
B

The nurse note a depressed female client has been more withdrawn and non-communicative during the past two weeks. Which intervention is most important to include in the updated plan of care for this client?

A.)Encourage the client’s family to visit more often

B.)Schedule a daily conference with the social worker

C.)Encourage the client to participate in group activities

D.)Engage the client in a non-threatening conversation.
D

A client with rheumatoid arthritis (RA) starts a new prescription of etanercept (Enbrel) subcutaneously once weekly. The nurse should emphasize the importance of reporting problem to the healthcare provider?

A.)Headache

B.)Joint stiffness

C.)Persistent fever

D.)Increase hunger and thirst
C

The nurse is assessing an older adult with type 2 diabetes mellitus. Which assessment finding indicates that the client understands long- term control of diabetes?

A.)The fating blood sugar was 120 mg/dl this morning.

B.)Urine ketones have been negative for the past 6 months

C.)The hemoglobin A1C was 6.5g/100 ml last week

D.)No diabetic ketoacidosis has occurred in 6 months.
C

An older male client is admitted with the medical diagnosis of possible cerebral vascular accident (CVA). He has facial paralysis and cannot move his left side. When entering the room, the nurse finds the client’s wife tearful and trying unsuccessfully to give him a drink of water. What action should the nurse take?
Ask the wife to stop and assess the client’s swallowing reflex

A 13 years-old client with non-union of a comminuted fracture of the tibia is admitted with osteomyelitis. The healthcare provider collects home aspirate specimens for culture and sensitivity and applies a cast to the adolescent’s lower leg. What action should the nurse implement next?

A.)Administer antiemetic agents

B.) Bivalve the cast for distal compromise

C.)Provide high- calorie, high-protein diet

D.)Begin parenteral antibiotic therapy
D

The nurse is preparing a community education program on osteoporosis. Which instruction is helpful in preventing bone loss and promoting bone formation?
Recommend weigh bearing physical activity

A client with a history of chronic pain requests a nonopioid analgesic. The client is alert but has difficulty describing the exact nature and location of the pain to the nurse. What action should the nurse implement next?
Administer the analgesic as requested

A male client receives a thrombolytic medication following a myocardial infarction. When the client has a bowel movement, what action should the nurse implement?

A.)Send stool sample to the lab for a guaiac test

B.)Observe stool for a day-colored appearance.

C.)Obtain specimen for culture and sensitivity analysis

D.)Asses for fatty yellow streaks in the client’s stool.
A

The mother of a child with cerebral palsy (CP) ask the nurse if her child’s impaired movements will worsen as the child grows. Which response provides the best explanation?
Brain damage with CP is not progressive but does have a variable course

During shift report, the central electrocardiogram (EKG) monitoring system alarms. Which client alarm should the nurse investigate first?
Respiratory apnea of 30 seconds

In early septic shock states, what is the primary cause of hypotension?

A.)Peripheral vasoconstriction

B.)Peripheral vasodilation

C.)Cardiac failure

D.)A vagal response
B

A client diagnosed with calcium kidney stones has a history of gout. A new prescription for aluminum hydroxide (Amphogel) is scheduled to begin at 0730. Which client medication should the nurse bring to the healthcare provider’s attention?

A.)Aspirin, low dose

B.)Furosemide (lasix)

C.)Enalapril (vasote)

D.)Allopurinol (Zyloprim)
D

A male client’s laboratory results include a platelet count of 105,000/ mm3 Based on this finding the nurse should include which action in the client’s plan of care?

A.)Cluster care to conserve
energy

B.)Initiate contact isolation

C.)Encourage him to use an electric razor

D.)Asses him for adventitious lung sounds
C

A client is admitted to the hospital after experiencing a brain attack, commonly referred to as a stroke or cerebral vascular accident (CVA). The nurse should request a referral for speech therapy if the client exhibits which finding?

A.)Abnormal responses for cranial nerves I and II

B.)Persistent coughing while drinking

C.)Unilateral facial drooping

D.)Inappropriate or exaggerated
mood swings
B

At 1615, prior to ambulating a postoperative client for the first time, the nurse reviews the client’s medical record. Based on date contained in the record, what action should the nurse take before assisting the client with ambulation:

A.)Remove sequential compression devices.

B.)Apply PRN oxygen per nasal cannula.

C.)Administer a PRN dose of an antipyretic.

D.)Reinforce the surgical wound dressing.
A

Which assessment finding for a client who is experiencing pontine myelinolysis should the nurse report to the healthcare provider?

A.)Sudden dysphagia

B.)Blurred visual field

C.)Gradual weakness

D.)Profuse diarrhea
A

A client is scheduled to receive an IW dose of ondansetron (Zofran) eight hours after receiving chemotherapy. The client has saline lock and is sleeping quietly without any restlessness. The nurse caring for the client is not certified in chemotherapy administration. What action should the nurse take?

A.)Ask a chemotherapy-certified nurse to administer the Zofran

B.)Administer the Zofran after flushing the saline lock with saline

C.)Hold the scheduled dose of Zofran until the client awakens

D.)Awaken the client to assess the need for administration of the Zofran.
B

When providing diet teaching for a client with cholecystitis, which types of food choices the nurse recommend to the client?

A.)High protein

B.)Low fat

C.)Low sodium

D.)High carbohydrate.
B

A client with a history of cirrhosis and alcoholism is admitted with severe dyspnea and ascites. Which assessment finding warrants immediate intervention by the nurse?

A.) Jaundice skin tone

B.)Muffled heart sounds

C.)Pitting peripheral edema

D.)Bilateral scleral edema
B

When entering a client’s room, the nurse discovers that the client is unresponsive and pulseless. The nurse initiate CPR and Calls for assistance. Which action should the nurse take next?

A.)Prepare to administer atropine 0.4 mg IVP

B.)Gather emergency tracheostomy equipment

C.)Prepare to administer lidocaine at 100 mg IVP

D.)Place cardiac monitor leads on the client’s chest.
D

A client with a history of dementia has become increasingly confused at night and is picking at an abdominal surgical dressing and the tape securing the intravenous (IV) line. The abdominal dressing is no longer occlusive, and the IV insertion site is pink. What intervention should the nurse implement?

A.)Replace the IV site with a smaller gauge.

B.)Redress the abdominal incision

C.)Leave the lights on in the room at night.

D.)Apply soft bilateral wrist restraints.
B

An adult male client is admitted to the emergency room following an automobile collision in which he sustained a head injury. What assessment data would provide the earliest that the client is experiencing increased intracranial pressure (ICP)?

A.)Lethargy

B.)Decorticate posturing

C.)Fixed dilated pupil

D.)Clear drainage from the ear.
A

In preparing a diabetes education program, which goal should the nurse identify as the primary emphasis for a class on diabetes self-management?

A.)Prepare the client to independently treat their disease process

B.)Reduce healthcare costs related to diabetic complications

C.)Enable clients to become active participating in controlling the disease process

D.)Increase client’s knowledge of the diabetic disease process and treatment options.
C

To reduce staff nurse role ambiguity, which strategy should the nurse manager implemented?

A.)Confirm that all the staff nurses are being assigned to equal number of clients.

B.)Review the staff nurse job description to ensure that it is clear, accurate, and recurrent.

C.)Assign each staff nurse a turn unit charge nurse on a regular, rotating basis.

D.)Analyze the amount of overtime needed by the nursing staff to complete assignments.
B

The nurse is assisting a new mother with infant feeding. Which information should the nurse provide that is most likely to result in a decrease milk supply for the mother who is breastfeeding?

A.)Supplemental feedings with formula

B.)Maternal diet high in protein

C.)Maternal intake of increased oral fluid

D.)Breastfeeding every 2 or 3 hours.
A

Which assessment is more important for the nurse to include in the daily plan of care for a client with a burned extremity?

A.)Range of Motion

B.)Distal pulse intensity

C.)Extremity sensation

D.)Presence of exudate
B

An elderly client with degenerative joint disease asks if she should use the rubber jar openers that are available. The nurse’s response should be based on which information about assistive devices?

When assessing a 6-month old infant, the nurse determines that the anterior fontanel is bulging. In which situation would this finding be most significant?

A.)Crying

B.)Straining on stool

C.)Vomiting

D.)Sitting upright.
D

A client with angina pectoris is being discharge from the hospital. What instruction should the nurse plan to include in this discharge teaching?

A.)Engage in physical exercise immediately after eating to help decrease cholesterol levels.

B.)Walk briskly in cold weather to increase cardiac output

C.)Keep nitroglycerin in a light-colored plastic bottle and readily available.

D.)Avoid all isometric exercises, but walk regularly.
D

What is the priority nursing action when initiating morphine therapy via an intravenous patient-controlled analgesia (PCA) pump?

A.)Initiate the dosage lockout mechanism on the PCA pump

B.)Instruct the client to use the medication before the pain becomes severe

C.)Assess the abdomen for bowel sounds.

D.)Assess the client ability to use a numeric pain scale
A

While undergoing hemodialysis, a male client suddenly complains of dizziness. He is alert and oriented, but his skin is cool and clammy. His vital signs are: heart rate 128 beats/minute, respirations 18 breaths/minute, and blood pressure 90/60. Which intervention should the nurse implement first?

The nurse receives a newborn within the first minutes after a vaginal delivery and intervenes to establish adequate respirations. What priority issue should the nurse address to ensure the newborn’s survival?

A.)Heat loss

B.)Hypoglycemia

C.)Fluid balance

D.)Bleeding tendencies
A

The fire alarm goes off while the charge nurse is receiving the shift report. What action should the charge nurse implement first?

A 60-year-old female client asks the nurse about hormones replacement therapy (HRT) as a means preventing osteoporosis. Which factor in the client’s history is a possible contraindication for the use of HRT?

A male client, who is 24 hours postoperative for an exploratory laparotomy, complains that he is “starving” because he has had no “real food” since before the surgery. Prior to advancing his diet, which intervention should the nurse implement?

The nurse working in the psychiatric clinic has phone messages from several clients. Which call should the nurse return first?

During change of shift, the nurse reports that a male client who had abdominal surgery yesterday increasingly confused and disoriented during the night. He wandered into other clients rooms, saying that there are men in his room trying to hurt him. Because of continuing disorientation and the client’s multiple attempts to get of bed, soft restrains were applied at 0400. In what order should the nurse who is receiving report implement these interventions? (Arrange from first action on top to last on the bottom).

Assign unlicensed assistive personnel to remove restrains and remain with client

Assess the client’s skin and circulation for impairment related to the restrains

Contact the client’s surgeon and primary healthcare provider

Evaluate the client’s mentation to determine need to continue the restrains

A mother brings her 3-year-old son to the emergency room and tells the nurse the he has had an upper respiratory infection for the past two days. Assessment of the child reveals a rectal temperature of 102 F. he is drooling and becoming increasingly more restless. What action should the nurse take first?

After receiving the first dose of penicillin, the client begins wheezing and has trouble breathing. The nurse notifies the healthcare provider immediately and received several prescriptions. Which medication prescription should the nurse administer first?

Two clients ring their call bells simultaneously requesting pain medication. What action should the nurse implement first?

A client receives a new prescription for simvastatin (Zocor) 5 mg PO daily at bedtime. What action should the nurse take?

Which client should the nurse assess frequently because of the risk for overflow incontinence? A client
Who is confused and frequently forgets to go to the bathroom

While monitoring a client during a seizure, which interventions should the nurse implement? (Select all that apply)

A.)Move obstacle away from client

B.)Monitor physical movements

C.)Observe for a patent airway

D.)Record the duration of the
seizure
A,B,C,D

A male client with a long history of alcoholism is admitted because of mild confusion and fine motor tremors. He reports that he quit drinking alcohol and stopped smoking cigarettes one month ago after his brother died of lung cancer. Which intervention is most important for the nurses to include in the client’s plan of care?

A.)Determine client’s level current blood alcohol level.

B.)Observe for changes in level of consciousness.

C.)Involve the client’s family in healthcare decisions.

D.)Provide grief counseling for client and his family.
B

An older adult female admitted to the intensive care unit (ICU) with a possible stroke is intubated with ventilator setting of tidal volume 600, PlO2 40%, and respiratory rate of 12 breaths/minute. The arterial blood gas (ABG) results after intubation are PH 7.31. PaCO2 60, PaO2 104, SPO2 98%, HCO3 23. To normalize the client’s ABG finding, which action is required?

A.)Report the results to the healthcare provider.

B.)Increase ventilator rate.

C.)Administer a dose of sodium carbonate.

D.)Decrease the flow rate of oxygen.
B

The mother of the 12- month-old with cystic fibrosis reports that her child is experiencing increasing congestion despite the use of chest physical therapy (CPT) twice a day, and has also experiences a loss of appetite. What instruction should the nurse provide?

A.)Perform CPT after meals to increase appetite and improve food intake.

B.)CPT should be performed more frequently, but at least an hour before meals.

C.)Stop using CPT during the daytime until the child has regained an appetite.

D.)Perform CPT only in the morning, but increase frequency when appetite improves.
B

The nurse is evaluating the diet teaching of a client with hypertension. What dinner selection indicates that the client understands the dietary recommendation for hypertension?

A client with type 2 diabetes mellitus is admitted for frequent hyperglycemic episodes and a glycosylated hemoglobin (HbA1c) of 10%. Insulin glargine 10 units subcutaneously once a day at bedtime and a sliding scale with insulin aspart q6h are prescribed. What action should the nurse include in this client’s plan of care?

A.)Fingerstick glucose assessment q6h with meals
Mix bedtime dose of insulin glargine with insulin aspart sliding scale dose

B.)Review with the client proper foot care and prevention of injury

C.)Do not contaminate the insulin aspart so that it is available for iv use

D.)Coordinate carbohydrate controlled meals at consistent times and intervals

E.)Teach subcutaneous injection
technique, site rotation and insulin management
A,B,D,E

Which problem reported by a client taking lovastatin requires the most immediate fallow up by the nurse?

A.)Diarrhea and flatulence

B.)Abdominal cramps

C.)Muscle pain

D.)Altered taste
C

While assessing a client’s chest tube (CT), the nurse discovers bubbling in the water seal chamber of the chest tube collection device. The client’s vital signs are: blood pressure of 80/40 mmHg, heart rate 120 beats/minutes, respiratory rate 32 breaths/minutes, oxygen saturation 88%. Which interventions should the nurse implement? SATA

A.)Provide supplemental oxygen

B.)Auscultate bilateral lung fields

C.)Administer a nebulizer treatment

D.)Reinforce occlusive CT dressing

E.)Give PRN dose of pain medication
A,B,D

Before leaving the room of a confused client, the nurse notes that a half bow knot was used to attach the client’s wrist restraints to the movable portion of the client’s bed frame. What action should the nurse take before leaving the room?

A.)Ensure that the knot can be quickly released.

B.)Tie the knot with a double turn or square knot.

C.)Move the ties so the restraints are secured to the side rails.

D.)Ensure that the restraints are snug against the client’s wrist.
A

Oral antibiotics are prescribed for an 18-month-old toddler with severe otitis media. An antipyrine and benzocaine-otic also prescribed for pain and inflammation. What instruction should the nurse emphasize concerning the installation of the antipyrine/benzocaine otic solution?

A.)Place the dropper on the upper outer ear canal and instill the medication slowly.

B.)Warm the medication in the microwave for 10 seconds before instilling.

C.)Keep the medication refrigerated between administrations.

D.)Have the child lie with the ear up for one to two minute after installation.
D

An older adult male is admitted with complications related to chronic obstructive pulmonary disease (COPD). He reports progressive dyspnea that worsens on exertion and his weakness has increased over the past month. The nurse notes that he has dependent edema in both lower legs. Based on these assessment findings, which dietary instruction should the nurse provide?

A.)Limit the intake of high calorie foods.

B.)Eat meals at the same time daily.

C.)Maintain a low protein diet.

D.)Restrict daily fluid intake.
D

The nurse inserts an indwelling urinary catheter as seen in the video what action should the nurse take next?

A.)Remove the catheter and insert into urethral opening

B.)Observe for urine flow and then inflate the balloon.

C.)Insert the catheter further and observe for discomfort.

D.)Leave the catheter in place and obtain a sterile catheter.
D

2018 HESI EXIT V61. A parent tells the nurse that their 6 year-old child who normally enjoys school, has notbeen doing well since the grandmother died 2 months ago. Which statement most accurately describes thoughts ondeath and dying at this age?A) Death is personified as the bogeyman or devilB) Death is perceived as being irreversibleC) The child feels guilty for the grandmother’s deathD) The child is worried that he, too, might dieThe correct answer is A: Death is personified as the bogeyman or devil2. A 67 year-old client with non-insulin dependent diabetes should be instructed tocontact the out-patient clinic immediately if the following findings are presentA) Temperature of 37.5 degrees Celsius with painful urinationB) An open wound on their heelC) Insomnia and daytime fatigueD) Nausea with 2 episodes of vomitingThe correct answer is B: An open wound on their heel3. The nurse admits an elderly Mexican-American migrant worker after an accident thatoccurred during work. To facilitate communication the nurse should initiallyA) Request a Spanish interpreterB) Speak through the family or co-workersC) Use pictures, letter boards, or monitoringD) Assess the client’s ability to speak EnglishThe correct answer is D: Assess the client”s ability to speak English4. In assessing a post partum client, the nurse palpates a firm fundus and observes a constant trickle of bright red blood from the vagina. What is the most likely cause ofthese findings?A) Uterine atonyB) Genital lacerationsC) Retained placentaD) Clotting disorderThe correct answer is B: Genital lacerations5. The nurse notes an abrupt onset of confusion in an elderly patient. Which of the following recently-ordered medications would most likely contribute to this change?
A) AnticoagulantB) Liquid antacidC) AntihistamineD) Cardiac glycosideThe correct answer is C: Antihistamine6. The nurse is caring for a client with active tuberculosis who has a history of noncompliance. Which of the following actions by the nurse would represent appropriatecare for this client?A) Instruct the client to wear a high efficiency particulate air mask in public places.B) Ask a family member to supervise daily complianceC) Schedule weekly clinic visits for the clientD) Ask the health care provider to change the regimen to fewer medications The correct answer is B: Ask a family member to supervise daily compliance7. The nurse manager identifies that time spent by staff in charting is excessive, requiringovertime for completion. The nurse manager states that “staff will form a task force to investigate and develop potential solutions to the problem, and report on this at the next staff meeting.” The nurse manager’s leadership style is best described asA) Laissez-faireB) AutocraticC) ParticipativeD) GroupThe correct answer is C: Participative8. A nursing student asks the nurse manager to explain the forces that drive health carereform. The appropriate response by the nurse manager should includeA) The escalation of fees with a decreased reimbursement percentageB) High costs of diagnostic and end-of-life treatment proceduresC) Increased numbers of elderly and of the chronically ill of all agesD) A steep rise in health care provider fees and in insurance premiumsThe correct answer is A: The escalation of fees with a decreased reimbursement percentage9. A client with hepatitis A (HAV) is newly admitted to the unit. Which action would bethe priority to include in the plan of care within the initial 24 hours for this client?A) Wear masks with shields if potential splashB) Use disposable utensils and plates for mealsC) Wear gown and gloves during client contactD) Provide soft easily digested food with frequent snacks
The correct answer is C: Wear gown and gloves during client contact10. A client has been taking alprazolam (Xanax) for 3 days. Nursing assessment shouldreveal which expected effect of the drug?A) Tranquilization, numbing of emotionsB) Sedation, analgesiaC) Relief of insomnia and phobiasD) Diminished tachycardia and tremors associated with anxietyThe correct answer is A: Tranquilization, numbing of emotions11. The nurse observes a staff member caring for a client with a left unilateralmastectomy. The nurse would intervene if she notices the staff member isA) Advising client to restrict sodium intakeB) Taking the blood pressure in the left armC) Elevating her left arm above heart levelD) Compressing the drainage deviceThe correct answer is B: Taking the blood pressure in the left arm12. A 70 year-old post-operative client has elevated serum BUN, Hct, Cl, and Na+.Creatinine and K+ are within normal limits. The nurse should perform additional assessments to confirm that an actualproblem is:A) Impaired gas exchangeB) Metabolic acidosisC) Renal insufficiencyD) Fluid volume deficitThe correct answer is D: Fluid volume deficit13. The nurse is providing foot care instructions to a client with arterial insufficiency. Thenurse would identify the need for additional teaching if the client statedA) “I can only wear cotton socks.”B) “I cannot go barefoot around my house.”C) “I will trim corns and calluses regularly.”D) “I should ask a family member to inspect my feet daily.”The correct answer is C: “I will trim corns and calluses regularly.”14. A woman who delivered 5 days ago and had been diagnosed with preeclampsia calls
the hospital triage nurse hotline to ask for advice. She states “ I have had the worst headache for the past 2 days. Itpounds and by the middle of the afternoon everything I look at looks wavy. Nothing I have taken helps.” What should the nurse do next?A) Advise the client that the swings in her hormones may have that effect. However,suggest for her to call her health care provider within the next day.B) Advise the client to have someone bring her to the emergency room as soon aspossibleC) Ask the client to stay on the line, get the address and send an ambulance to the homeD) Ask what the client has taken? How often? Ask about other specific complaints. The correct answer is C: Ask the client to stay on the line, get the address and send anambulance to the home15. The primary teaching for a client following an extracorporeal shock-wave lithotripsy(ESWL) procedure isA) Drink 3000 to 4000 cc of fluid each day for one monthB) Limit fluid intake to 1000 cc each day for one monthC) Increase intake of citrus fruits to three servings per dayD) Restrict milk and dairy products for one monthThe correct answer is A: Drink 3000 to 4000 cc of fluid each day for 1 month16. A client on warfarin therapy following coronary artery stent placement calls the clinicto ask if he can take Alka-Seltzer for an upset stomach. What is the best response by the nurse?A) Avoid Alka-Seltzer because it contains aspirinB) Take Alka-Seltzer at a different time of day than the warfarinC) Select another antacid that does not inactivate warfarinD) Use on-half the recommended dose of Alka-SeltzerThe correct answer is A: Avoid Alka-Seltzer because it contains aspirin17. The nurse is working with parents to plan home care for a 2 year-old with a heartproblem. A priority nursing intervention would be toA) Encourage the parents to enroll in cardiopulmonary resuscitation classB) Assist the parents to plan quiet play activities at homeC) Stress to the parents that they will need relief care giversD) Instruct the parents to avoid contact with persons with infectionThe correct answer is A: Encourage the parents to enroll in cardiopulmonary resuscitation class

  1. The nurse is caring for a client with Rheumatoid Arthritis. Which nursing diagnosisshould receive priority in the plan of care?A) Risk for injuryB) Self care deficitC) Alteration in comfortD) Alteration in mobilityThe correct answer is C: Alteration in comfort19. An unlicensed assistive staff member asks the nurse manager to explain the beliefs ofa Christian Scientist who refuses admission to the hospital after a motor vehicle accident.The best response of the nurse would be which of these statements?A) “Spiritual healing is emphasized and the mind contributes to the cure.”B) “The primary belief is that dietary practices result in health or illness.”C) “Fasting and prayer are initial actions to take in physical injury.”D) “Meditation is intensive in the initial 48 hours and daily thereafter.”The correct answer is A: “Spiritual healing is emphasized and the mind contributes to the cure.”20. In order to be effective in administering cardiopulmonary resuscitation to a 5 year-old, the nurse mustA) Assess the brachial pulsesB) Breathe once every 5 compressionsC) Use both hands to apply chest pressureD) Compress 80-90 times per minuteThe correct answer is B: Breathe once every 5 compressions21. The nurse is providing home care for a client with heart failure and pulmonary edema.Which nursing diagnosis should have priority in planning care?A) Impaired skin integrity related to dependent edemaB) Activity intolerance related to oxygen supply and demand imbalanceC) Constipation related to immobilityD) Risk for infection related to ineffective mobilization of secretionsThe correct answer is B: Activity Intolerance related to oxygen supply and demand imbalance.22. For which of the following mother-baby pairs should the nurse review the Coomb’s test in preparation for administering RhO (D) immune globulin within 72 hours of birth?A) Rh negative mother with Rh positive babyB) Rh negative mother with Rh negative baby
    C) Rh positive mother with Rh positive babyD) Rh positive mother with Rh negative babyThe correct answer is A: Rh negative mother with Rh positive baby23. An 80 year-old nursing home resident has a temperature of 101.6 degrees Fahrenheitrectally. This is a sudden change in an otherwise healthy client. Which should the nurse assess first?A) Lung soundsB) Urine outputC) Level of alertnessD) AppetiteThe correct answer is C: Level of alertness?25. What is the major purpose of community health research?A) Describe the health conditions of populationsB) Evaluate illness in the communityC) Explain the health conditions of familiesD) Identify the health conditions of the environmentThe correct answer is A: Describe the health conditions of populations26. The recent increase in the reported cases of active tuberculosis (TB) in the UnitedStates is attributed to which factor?A) The increased homeless population in major citiesB) The rise in reported cases of positive HIV infectionsC) The migration patterns of people from foreign countriesD) The aging of the population located in group homesThe correct answer is B: The rise in reported cases of positive HIV infections27. A 15 month-old child comes to the clinic for a follow-up visit after hospitalization fortreatment of Kawasaki Disease. The nurse recognizes that which of the following scheduled immunizations will bedelayed?A) MMRB) HibC) IPVD) DtaPThe correct answer is A: MMR

Document continues below
Discover more from:
Community health nursing (NUR 2611)(NUR 2611)
396 documents
Go to course
34
AORN Periop 101 Final EXAM Q
AORN Periop 101 Final EXAM Q
Community health nursing
100% (20)
20
Med Surg ATI Proctored Exam Test Bank A 1
Med Surg ATI Proctored Exam Test Bank A 1
Community health nursing
96% (25)
7
Basic Life Support Exam A
Basic Life Support Exam A
Community health nursing
100% (7)
15
ATI RN MEDSURG 2021/2022 PROCTORED EXAM- LATEST 100% CORRECT STUDY GUIDE.Q$A WITH RATIONALES.
ATI RN MEDSURG 2021/2022 PROCTORED EXAM- LATEST 100% CORRECT STUDY GUIDE.Q$A WITH RATIONALES.
Community health nursing
100% (7)
12
2022 NRP Practice EXAM Questions AND Answers ALL Solved Solution
2022 NRP Practice EXAM Questions AND Answers ALL Solved Solution
Community health nursing
89% (19)
27
Sherman Red Yoder All Parts
Sherman Red Yoder All Parts
Community health nursing
100% (4)

  1. The nurse is assessing a pregnant client in her third trimester. The parents are informed that the ultrasound suggests that the baby is small for gestational age (SGA). Anearlier ultrasound indicated normal growth. The nurse understands that this change is most likely due to what factor?A) Sexually transmitted infectionB) Exposure to teratogensC) Maternal hypertensionD) Chromosomal abnormalitiesThe correct answer is C: Maternal hypertension29. After the shift report in a labor and delivery unit which of these clients would thenurse check first?A) A middle aged woman with asthma and diabetes mellitus Type 1 has a BP of 150/94B) A middle aged woman with a history of two prior vaginal term births is 2 cm dilatedC) A young woman is a grand multipara has cervical dilation of 4 cm and 50% effacedD) An adolescent who is 18 weeks pregnant has a report of no fetal heart tones andcoughing up frothy sputumThe correct answer is D: An adolescent who is 18 weeks pregnant has a report of no fetal heart tones and coughing up frothy sputum30. The nurse is caring for an 87 year-old client with urinary retention. Which findingshould be reported immediately?A) Fecal impactionB) Infrequent voidingC) Stress incontinenceD) Burning with urinationThe correct answer is A: Fecal impaction31. The nasogastric tube of a post-op gastrectomy client has stopped draining greenishliquid. The nurse shouldA) Irrigate it as ordered with distilled waterB) Irrigate it as ordered with normal salineC) Place the end of the tube in water to see if the water bubblesD) Withdraw the tube several inches and reposition itThe correct answer is B: Irrigate it as ordered with normal saline32. The parents of a child who has recently been diagnosed with asthma ask the nurse to
    explain the condition to them. The best response is “Asthma causes…A) the airway to become narrow and obstructs airflow.”B) air to be trapped in the lungs because the airways are dilated.”C) the nerves that control respiration to become hyperactive.”D) a decrease in the stress hormones which prevents the airways from opening.”The correct answer is A: the airway to become narrow and obstructs airflow.”33. The nurse is assessing a child with suspected lead poisoning. Which of the followingassessments is the nurse most likely to find?A) Complaints of numbness and tingling in feetB) Wheezing noted when lung sound auscultatedC) Excessive perspirationD) Difficulty sleepingThe correct answer is A: Complaints of numbness and tingling in feet34. The nurse is caring for a client with end-stage heart failure. The family members aredistressed about the client’s impending death. What action should the nurse do first?A) Explain the stages of death and dying to the familyB) Recommend an easy-to-read book on griefC) Assess the family’s patterns for dealing with deathD) Ask about their religious affiliationsThe correct answer is C: Assess the family”s patterns for dealing with death35. The nurse is caring for a client with Meniere’s disease. When teaching the client aboutthe disease, the nurse should explain that the client should avoid foods high inA) CalciumB) FiberC) SodiumD) CarbohydrateThe correct answer is C: Sodium36. The nurse is teaching a mother who will breast feed for the first time. Which of thefollowing is a priority?A) Show her films on the physiology of lactationB) Give the client several illustrated pamphletsC) Assist her to position the newborn at the breastD) Give her privacy for the initial feedingThe correct answer is C: Assist her to position the newborn at the breast
  2. The nurse is taking a health history from parents of a child admitted with possible Reye’s Syndrome. Which recent illness would the nurse recognize as increasing the riskto develop Reye’sSyndrome?A) RubeolaB) MeningitisC) VaricellaD) HepatitisThe correct answer is C: Varicella38. While giving care to a 2 year-old client, the nurse should remember that the toddler’stendency to say “no” to almost everything is an indication of what psychosocial skill?A) Stubborn behaviorB) Rejection of parentsC) Frustration with adultsD) Assertion of controlThe correct answer is D: Assertion of control39. A postpartum client admits to alcohol use throughout the pregnancy. Which of thefollowing newborn assessments suggests to the nurse that the infant has fetal alcohol syndrome?A) Growth retardation is evidentB) Multiple anomalies are identifiedC) Cranial facial abnormalities are notedD) Prune belly syndrome is suspectedThe correct answer is C: Cranial facial abnormalities are noted40. The nurse is attending a workshop about caring for persons infected with Hepatitis.Which statement is correct when referring to the incidence rate for Hepatitis?A) The number of persons in a population who develop Hepatitis B during a specificperiod of timeB) The total number of persons in a population who have Hepatitis B at a particular timeC) The percentage of deaths resulting from Hepatitis B during a specific timeD) The occurrence of Hepatitis B in the population at a particular timeThe correct answer is A: The number of persons in a population who develop Hepatitis B during a specific period of time
  3. A 36 year-old female client has a hemoglobin level of 14 g/dl and a hematocrit of42% following a D&C. Which of the following would the nurse expect to find whenassessing this client?A) Capillary refill less than 3 secondsB) Pale mucous membranesC) Respirations 36 breaths per minuteD) Complaints of fatigue when ambulatingThe correct answer is A: Capillary refill less than 3 seconds42. The nurse is caring for a client suspected to have Tuberculosis (TB). Which of the following diagnostic tests is essential for determining the presence of active TB? The nurse is caring for a client suspected to have Tuberculosis (TB). Which of the followingdiagnostic tests is essential for determining the presence of active TB?A) Tuberculin skin testingB) Sputum cultureC) White blood cell countD) Chest x-rayThe correct answer is B: Sputum culture43. The nurse has been teaching an apprehensive primipara who has difficulty in initialnursing of the newborn. What observation at the time of discharge suggests that initial breast feeding is effective?A) The mother feels calmer and talks to the baby while nursingB) The mother awakens the newborn to feed whenever it falls asleepC) The newborn falls asleep after 3 minutes at the breastD) The newborn refuses the supplemental bottle of glucose waterThe correct answer is A: The mother feels calmer and talks to the baby while nursing44. The mother of a burned child asks the nurse to clarify what is meant by a third degreeburn. The best response by the nurse isA) “The top layer of the skin is destroyed.”B) “The skin layers are swollen and reddened.”C) “All layers of the skin were destroyed in the burn.”D) “Muscle, tissue and bone have been injured.”The correct answer is C: “All layers of the skin were destroyed in the burn.”45. The nurse is taking a health history from a Native American client. It is critical thatthe nurse must remember that eye contact with such clients is considered
    A) ExpectedB) RudeC) ProfessionalD) EnjoyableThe correct answer is B: Rude46. A nurse is instructing a class for new parents at a local community center. The nursewould stress that which activity is most hazardous for an 8 month-old child?A) Riding in a carB) Falling off a bedC) Electrical outletsD) Eating peanutsThe correct answer is D: Eating peanuts47. When teaching parents about sickle cell disease, the nurse should tell them that theirchild’s anemia is caused byA) Reduced oxygen capacity of cells due to lack of ironB) An imbalance between red cell destruction and productionC) Depression of red and white cells and plateletsD) Inability of sickle shaped cells to regenerateThe correct answer is B: An imbalance between red cell destruction and production48. The nurse is assessing a newborn delivered at home by an admitted heroin addict.Which of the following would the nurse expect to observe?A) Hypertonic neuro reflexB) Immediate CNS depressionC) Lethargy and sleepinessD) Jitteriness at 24-48 hoursThe correct answer is D: Jitteriness at 24-48 hours49. The nurse is caring for a client with congestive heart failure. Which finding requiresthe nurse’s immediate attention?A) Pulse oximetry of 85%B) NocturiaC) Crackles in lungsD) DiaphoresisThe correct answer is A: Pulse oximetry of 85%
  4. The nurse is assessing a young child at a clinic visit for a mild respiratory infection.Koplik spots are noted on the oral mucous membranes. The nurse should then assess which area of the body?A) Inspect the skinB) Auscultate breath soundsC) Evaluate muscle strengthD) Investigate elimination patternsThe correct answer is A: Inspect the skin51. Which action is most likely to ensure the safety of the nurse while making a homevisit?A) Observation during the visit of no evidence of weapons in the homeB) Prior to the visit, review client’s record for any previous entries about violenceC) Remain alert at all times and leave if cues suggest the home is not safeD) Carry a cell phone, pager and/or hand held alarm for emergenciesThe correct answer is C: Staying alert at all times and leaving if cues suggest the home is not safe52. An adolescent client is admitted in respiratory alkalosis following aspirin overdose.The nurse recognizes that this imbalance was caused byA) TachypneaB) Acidic byproductsC) Vomiting and dehydrationD) HyperpyrexiaThe correct answer is A: Tachypnea53. The nurse discovers that the parents of a 2 year-old child continue to use an apnea monitor each night. The parents state: “We are concerned about the possible occurrenceof sudden infant deathsyndrome (SIDS).” In order to take appropriate action, the nurse must understand thatA) The child is within the age group most susceptible to SIDSB) The peak age for occurrence of SIDS is 8 to 12 months of ageC) The apnea monitor is not effective on a child in this age groupD) 95% of SIDS cases occur before 6 months of ageThe correct answer is D: 95% percent of all SIDS cases occur before 6 months54. As a client is being discharged following resolution of a spontaneous pneumothorax,
    he tells the nurse that he is now going to Hawaii for a vacation. The nurse would warnhim to avoidA) SurfingB) Scuba divingC) ParasailingD) SwimmingReview Information: The correct answer is B: Scuba diving55. The nurse is providing diet instruction to the parents of a child with cystic fibrosis.The nurse would emphasize that the diet should beA) High calorie, low fat, low sodiumB) High protein, low fat, low carbohydrateC) High protein, high calorie, unrestricted fatD) High carbohydrate, low protein, moderate fatThe correct answer is C: High protein, high calorie, unrestricted fat56. A client had arrived in the USA from a developing country 1 week prior. The client isto be admitted to the medical surgical unit with a diagnosis of AIDS with a history of unintended weight loss, drug abuse, night sweats, productive cough and a “feeling of being hot all the time.” The nurse should assign the client to share a room with a client with the diagnosis ofA) Acute tuberculosis with a productive cough of discolored sputum for over threemonthsB) Lupus and vesicles on one side of the middle trunk from the back to the abdomenC) Pseudomembranous colitis and C. difficile.D) Exacerbation of polyarthritis with severe painThe correct answer is A: Acute tuberculosis with a productive cough of discolored sputum for over three months57. A client’s admission urinalysis shows the specific gravity value of 1.039. Which of thefollowing assessment data would the nurse expect to find when assessing this client?A) Moist mucous membranesB) Urinary frequencyC) Poor skin turgorD) Increased blood pressureThe correct answer is C: Poor skin turgor58. Parents are concerned that their 11 year-old child is a very picky eater. The nurse
    suggests which of the following as the best initial approach?A) Consider a liquid supplement to increase caloriesB) Discuss consequences of an unbalanced diet with the childC) Provide fruit, vegetable and protein snacksD) Encourage the child to keep a daily log of foods eatenThe correct answer is B: Discuss consequences of an unbalanced diet with the child59. At a community health fair the blood pressure of a 62 year-old client is 160/96. Theclient states “My blood pressure is usually much lower.” The nurse should tell the clienttoA) go get a blood pressure check within the next 48 to 72 hoursB) check blood pressure again in 2 monthsC) see the health care provider immediatelyD) visit the health care provider within 1 week for a BP checkThe correct answer is A: go get a blood pressure check within the next 48 to 72 hours60. A client is admitted to the emergency room with renal calculi and is complaining ofmoderate to severe flank pain and nausea. The client’s temperature is 100.8 degrees Fahrenheit. The priority nursing goal for this client isA) Maintain fluid and electrolyte balanceB) Control nauseaC) Manage painD) Prevent urinary tract infectionThe correct answer is C: Manage pain The immediate goal of therapy is to alleviate the client’s pain.61. An RN who usually works in a spinal rehabilitation unit is floated to the emergencydepartment. Which of these clients should the charge nurse assign to this RN?A) A middle-aged client who says “I took too many diet pills” and “my heart feels like itis racing out of my chest.”B) A young adult who says “I hear songs from heaven. I need money for beer. I quitdrinking 2 days ago for my family. Why are my arms and legs jerking?”*C)An adolescent who has been on pain medications for terminal cancer with an initialassessment finding of pinpoint pupils and a relaxed respiratory rate of 10D) An elderly client who reports having taken a “large crack hit” 10 minutes prior to walking into the emergency roomThe correct answer is c: An adolescent who has been on pain medications for terminal cancer with an initial assessment finding of pinpoint pupils and a relaxed respiratory rate of 10
  5. While planning care for a toddler, the nurse teaches the parents about the expecteddevelopmental changes for this age. Which statement by the mother shows that she understands the child’s developmental needs?A) “I want to protect my child from any falls.”B) “I will set limits on exploring the house.”C) “I understand the need to use those new skills.”D) “I intend to keep control over our child.”The correct answer is C: “I understand the need to use those new skills.”63. A client who is pregnant comes to the clinic for a first visit. The nurse gathers data about her obstetric history, which includes 3 year old twins at home and a miscarriage 10years ago at 12 weeksgestation. How would the nurse accurately document this information?A) Gravida 4 para 2B) Gravida 2 para 1C) Gravida 3 para 1D) Gravida 3 para 2The correct answer is C: Gravida 3 para 164. A child who ingested 15 maximum strength acetaminophen tablets 45 minutes ago isseen in the emergency department. Which of these orders should the nurse do first?A) Gastric lavage PRNB) Acetylcysteine (mucomyst) for age per pharmacyC) Start an IV Dextrose 5% with 0.33% normal saline to keep vein openD) Activated charcoal per pharmacyThe correct answer is A: Gastric lavage PRN65. The nurse is preparing to administer an enteral feeding to a client via a nasogastricfeeding tube. The most important action of the nurse isA) Verify correct placement of the tubeB) Check that the feeding solution matches the dietary orderC) Aspirate abdominal contents to determine the amount of last feeding remaining instomachD) Ensure that feeding solution is at room temperatureThe correct answer is A: Verify correct placement of the tube66. The nurse anticipates that for a family who practices Chinese medicine the prioritygoal would be toA) Achieve harmony
    B) Maintain a balance of energyC) Respect lifeD) Restore yin and yangThe correct answer is D: Restore yin and yang67. Which complication of cardiac catheterization should the nurse monitor for in theinitial 24 hours after the procedure?A) angina at restB) thrombus formationC) dizzinessD) falling blood pressureThe correct answer is B: thrombus formation68. A nurse prepares to care for a 4 year-old newly admitted for rhabdomyosarcoma. Thenurse should alert the staff to pay more attention to the function of which area of the body?A) The musclesB) The cerebellumC) The kidneysD) The leg bonesThe correct answer is A: All striated muscles69. A client comes to the clinic for treatment of recurrent pelvic inflammatory disease. The nurse recognizes that this condition most frequently follows which type of infection?A) TrichomoniasisB) ChlamydiaC) StaphylococcusD) StreptococcusThe correct answer is B: Chlamydia70. During the evaluation of the quality of home care for a client with Alzheimer’s disease, the priority for the nurse is to reinforce which statement by a family member?A) At least 2 full meals a day is eaten.B) We go to a group discussion every week at our community center.C) We have safety bars installed in the bathroom and have 24 hour alarms on the doors.D) The medication is not a problem to have it taken 3 times a day.The correct answer is C: We have safety bars installed in the bathroom and have 24 hour alarms on the doors.
  6. The nurse is caring for a client with a venous stasis ulcer. Which nursing interventionwould be most effective in promoting healing?A) Apply dressing using sterile techniqueB) Improve the client’s nutrition statusC) Initiate limb compression therapyD) Begin proteolytic debridementThe correct answer is B: Improve the client”s nutrition status72. During an assessment of a client with cardiomyopathy, the nurse finds that the systolic blood pressure has decreased from 145 to 110 mm Hg and the heart rate has risenfrom 72 to 96 beats per minute and the client complains of periodic dizzy spells. The nurse instructs the client toA) Increase fluids that are high in proteinB) Restrict fluidsC) Force fluids and reassess blood pressureD) Limit fluids to non-caffeine beveragesThe correct answer is C: Force fluids and reassess blood pressure73. Which individual is at greatest risk for developing hypertension?A) 45 year-old African American attorneyB) 60 year-old Asian American shop ownerC) 40 year-old Caucasian nurseD) 55 year-old Hispanic teacherThe correct answer is A: 45 year-old African American attorney74. The nurse is caring for a client with a serum potassium level of 3.5 mEq/L. The clientis placed on a cardiac monitor and receives 40 mEq KCL in 1000 ml of 5% dextrose in water IV. Which of the following EKG patterns indicates to the nurse that the infusions should be discontinued?A) Narrowed QRS complexB) Shortened “PR” intervalC) Tall peaked T wavesD) Prominent “U” wavesThe correct answer is C: Tall peaked T waves75. A client has been taking furosemide (Lasix) for the past week. The nurse recognizeswhich finding may indicate the client is experiencing a negative side effect from the medication?
    A) Weight gain of 5 poundsB) Edema of the anklesC) Gastric irritabilityD) Decreased appetiteThe correct answer is D: Decreased appetite76. Which of these statements best describes the characteristic of an effective reward-feedback system?A) Specific feedback is given as close to the event as possibleB) Staff are given feedback in equal amounts over timeC) Positive statements are to precede a negative statementD) Performance goals should be higher than what is attainableThe correct answer is A: Specific feedback is given as close to the event as possible77. The nurse practicing in a maternity setting recognizes that the post mature fetus is atrisk due toA) Excessive fetal weightB) Low blood sugar levelsC) Depletion of subcutaneous fatD) Progressive placental insufficiencyThe correct answer is D: Progressive placental insufficiency78. A child who has recently been diagnosed with cystic fibrosis is in a pediatric clinic where a nurse is performing an assessment. Which later finding of this disease would thenurse not expect to see at thistime?A) Positive sweat testB) Bulky greasy stoolsC) Moist, productive coughD) Meconium ileusThe correct answer is C:Moist, productive cough79. Which of the following should the nurse implement to prepare a client for a KUB(Kidney, Ureter, Bladder) radiograph test?A) Client must be NPO before the examinationB) Enema to be administered prior to the examinationC) Medicate client with Lasix 20 mg IV 30 minutes prior to the examinationD) No special orders are necessary for this examination
    The correct answer is D: No special orders are necessary for this examination80. The hospital has sounded the call for a disaster drill on the evening shift. Which of these clients would the nurse put first on the list to be discharged in order to make a roomavailable for a new admission?A)A middle aged client with a history of being ventilator dependent for over 7 years andadmitted with bacterial pneumonia five days agoB) A young adult with diabetes mellitus Type 2 for over 10 years and admitted withantibiotic induced diarrhea 24 hours agoC) An elderly client with a history of hypertension, hypercholesterolemia and lupus, andwas admitted with Stevens- Johnson syndrome that morningD) An adolescent with a positive HIV test and admitted for acute cellulitus of the lowerleg 48 hours agoThe correct answer is A: A middle aged client with a history of being ventilator dependent for over 7 years and admitted with bacterial pneumonia five days ago81. A triage nurse has these 4 clients arrive in the emergency department within 15minutes. Which client should the triage nurse send back to be seen first?A) A 2 month old infant with a history of rolling off the bed and has bulging fontanelswith cryingB) A teenager who got a singed beard while campingC) An elderly client with complaints of frequent liquid brown colored stoolsD) A middle aged client with intermittent pain behind the right scapulaThe correct answer is B: A teenager who got singed a singed beard while camping82. A client is receiving digoxin (Lanoxin) 0.25 mg. Daily. The health care provider haswritten a new order to give metoprolol (Lopressor) 25 mg. B.I.D. In assessing the clientprior to administering the medications, which of the following should the nurse report immediately to the health care provider?A) Blood pressure 94/60B) Heart rate 76C) Urine output 50 ml/hourD) Respiratory rate 16The correct answer is A: Blood pressure 94/6083. A nurse enters a client’s room to discover that the client has no pulse or respirations.After calling for help, the first action the nurse should take isA) Start a peripheral IV
    B) Initiate closed-chest massageC) Establish an airwayD) Obtain the crash cartThe correct answer is C: Establish an airway84. A 3 year-old child comes to the pediatric clinic after the sudden onset of findings thatinclude irritability, thick muffled voice, croaking on inspiration, hot to touch, sit leaning forward, tongue protruding, drooling and suprasternal retractions. What should the nursedo first?A) Prepare the child for x-ray of upper airwaysB) Examine the child’s throatC) Collect a sputum specimenD) Notify the healthcare provider of the child’s statusThe correct answer is D: Notify the health care provider of the child”s status85. A nurse is to administer meperidine hydrochloride (Demerol) 100 mg, atropine sulfate(Atropisol) 0.4 mg, and promethizine hydrochloride (Phenergan) 50 mg IM to a pre- operative client. Which action should the nurse take first?A) Raise the side rails on the bedB) Place the call bell within reachC) Instruct the client to remain in bedD) Have the client empty bladderThe correct answer is D: Have the client empty bladder86. In children suspected to have a diagnosis of diabetes, which one of the followingcomplaints would be most likely to prompt parents to take their school age child forevaluation?A) PolyphagiaB) DehydrationC) Bed wettingD) Weight lossThe correct answer is C: Bed wetting87. A client has been newly diagnosed with hypothyroidism and will take levothyroxine (Synthroid) 50 mcg/day by mouth. As part of the teaching plan, the nurse emphasizes thatthis medication:A) Should be taken in the morningB) May decrease the client’s energy level
    C) Must be stored in a dark containerD) Will decrease the client’s heart rateThe correct answer is A: Should be taken in the morning88. A client has a Swan-Ganz catheter in place. The nurse understands that this isintended to measureA) Right heart functionB) Left heart functionC) Renal tubule functionD) Carotid artery functionThe correct answer is B: Left heart function89. Which of these findings indicate that a pump to deliver a basal rate of 10 ml per hourplus PRN for pain break through for morphine drip is not working?A) The client complains of discomfort at the IV insertion siteB) The client states “I just can’t get relief from my pain.”C) The level of drug is 100 ml at 8 AM and is 80 ml at noonD) The level of the drug is 100 ml at 8 AM and is 50 ml at noonThe correct answer is C: The level of drug is 100 ml at 8 AM and is 80 ml at noon90. The nurse is performing a neurological assessment on a client post right CVA. Whichfinding, if observed by the nurse, would warrant immediate attention?A) Decrease in level of consciousnessB) Loss of bladder controlC) Altered sensation to stimuliD) Emotional labilityThe correct answer is A: Decrease in level of consciousness91. When teaching a client with coronary artery disease about nutrition, the nurse shouldemphasizeA) Eating 3 balanced meals a dayB) Adding complex carbohydratesC) Avoiding very heavy mealsD) Limiting sodium to 7 gms per dayThe correct answer is C: Avoiding very heavy meals92. The nurse is speaking at a community meeting about personal responsibility for health promotion. A participant asks about chiropractic treatment for illnesses. What
    should be the focus of the nurse’s response?A) Electrical energy fieldsB) Spinal column manipulationC) Mind-body balanceD) Exercise of jointsThe correct answer is B: Spinal column manipulation93. The home health nurse visits a male client to provide wound care and finds the clientlethargic and confused. His wife states he fell down the stairs 2 hours ago. The nurse shouldA) Place a call to the client’s health care provider for instructionsB) Send him to the emergency room for evaluationC) Reassure the client’s wife that the symptoms are transientD) Instruct the client’s wife to call the doctor if his symptoms become worseThe correct answer is B: Send him to the emergency room for evaluation94. While assessing a 1 month-old infant, which finding should the nurse reportimmediately?A) Abdominal respirationsB) Irregular breathing rateC) Inspiratory gruntD) Increased heart rate with cryingReview Information: The correct answer is C: Inspiratory grunt95. A client with multiple sclerosis plans to begin an exercise program. In addition to discussing the benefits of regular exercise, the nurse should caution the client to avoidactivities whichA) Increase the heart rateB) Lead to dehydrationC) Are considered aerobicD) May be competitiveThe correct answer is B: Lead to dehydration96. The nurse is caring for a client who had a total hip replacement 4 days ago. Whichassessment requires the nurse’s immediate attention?A) I have bad muscle spasms in my lower leg of the affected extremity.B) “I just can’t ‘catch my breath’ over the past few minutes and I think I am in gravedanger.”
    C) “I have to use the bedpan to pass my water at least every 1 to 2 hours.”D) “It seems that the pain medication is not working as well today.”The correct answer is B: “I just can”t ”catch my breath” over the past few minutes and I think I am in grave danger.”97. The nurse is giving discharge teaching to a client 7 days post myocardial infarction.He asks the nurse why he must wait 6 weeks before having sexual intercourse. What isthe best response by the nurse to this question?A) “You need to regain your strength before attempting such exertion.”B) “When you can climb 2 flights of stairs without problems, it is generally safe.”C) “Have a glass of wine to relax you, then you can try to have sex.”D) “If you can maintain an active walking program, you will have less risk.”The correct answer is B: “When you can climb 2 flights of stairs without problems, it is generally safe.”98. What would the nurse expect to see while assessing the growth of children duringtheir school age years?A) Decreasing amounts of body fat and muscle massB) Little change in body appearance from year to yearC) Progressive height increase of 4 inches each yearD) Yearly weight gain of about 5.5 pounds per yearThe correct answer is D: Yearly weight gain of about 5.5 pounds per year99. The nurse is assigned to care for a client who has a leaking intracranial aneurysm. Tominimize the risk of rebleeding , the nurse should plan toA) Restrict visitors to immediate familyB) Avoid arousal of the client except for family visitsC) Keep client’s hips flexed at no less than 90 degreesD) Apply a warming blanket for temperatures of 98 degrees Fahrenheit or lessThe correct answer is A: Restrict visitors to immediate family100. The nurse is performing a gestational age assessment on a newborn delivered 2 hours ago. When comparing findings to the Ballard scale, which situation may affect thescore?A) Birth weightB) Racial differencesC) Fetal distress in laborD) Birth traumaThe correct answer is C: Fetal distress in labor
  7. A 4 year-old hospitalized child begins to have a seizure while playing with hard plastic toys in the hallway. Of the following nursing actions, which one should the nursedo first?A) Place the child in the nearest bedB) Administer IV medication to slow down the seizureC) Place a padded tongue blade in the child’s mouthD) Remove the child’s toys from the immediate areaThe correct answer is D: Remove the child’’s toys from the immediate area102. A client asks the nurse to explain the basic ideas of homeopathic medicine. Theresponse that best explains this approach is that remediesA) Destroy organisms causing diseaseB) Maintain fluid balanceC) Boost the immune systemD) Increase bodily energyThe correct answer is C: Boost the immune system103. The nurse is caring for a 2 year-old who is being treated with chelation therapy, calcium disodium edetate, for lead poisoning. The nurse should be alert for which of thefollowing side effects?A) NeurotoxicityB) HepatomegalyC) NephrotoxicityD) OtotoxicityThe correct answer is C: Nephrotoxicity104. The nurse is caring for a 1 year-old child who has 6 teeth. What is the best way forthe nurse to give mouth care to this child?A) Using a moist soft brush or cloth to clean teeth and gumsB) Swabbing teeth and gums with flavored mouthwashC) Offering a bottle of water for the child to drinkD) Brushing with toothpaste and flossing each toothThe correct answer is A: Using a moist soft brush or cloth to clean teeth and gums105. At a senior citizens meeting a nurse talks with a client who has diabetes mellitusType 1. Which statement by the client during the conversation is most predictive of apotential for impaired skinintegrity?
    A) “I give my insulin to myself in my thighs.”B) “Sometimes when I put my shoes on I don’t know where my toes are.”C) “Here are my up and down glucose readings that I wrote on my calendar.”D) “If I bathe more than once a week my skin feels too dry.”The correct answer is B: “Sometimes when I put my shoes on I don”t know where mytoes are.”106. A couple trying to conceive asks the nurse when ovulation occurs. The womanreports a regular 32 day cycle. Which response by the nurse is correct?A) Days 7-10B) Days 10-13C) Days 14-16D) Days 17-19The correct answer is D: Days 17-19107. Included in teaching the client with tuberculosis taking INH about follow-up homecare, the nurse should emphasize that a laboratory appointment for which of the following lab tests is critical?A) Liver functionB) Kidney functionC) Blood sugarD) Cardiac enzymesThe correct answer is A: Liver function108. A 78 year-old client with pneumonia has a productive cough but is confused. Safetyprotective devices (restraints) have been ordered for this client. How can the nurse prevent aspiration?A) Suction the client frequently while restrainedB) Secure all 4 restraints to 1 side of bedC) Obtain a sitter for the client while restrainedD) Request an order for a cough suppressantThe correct answer is C: Obtain a sitter for the client while restrained109. A client with a fractured femur has been in Russell’s traction for 24 hours. Whichnursing action is associated with this therapy?A) Check the skin on the sacrum for breakdownB) Inspect the pin site for signs of infectionC) Auscultate the lungs for atelectasis
    D) Perform a neurovascular check for circulationThe correct answer is D: Perform a neurovascular check for circulation110. The nurse is caring for a client with extracellular fluid volume deficit. Which ofthe following assessments would the nurse anticipate finding?A) Bounding pulseB) Rapid respirationsC) OliguriaD) Neck veins are distendedThe correct answer is C: Oliguria111. When suctioning a client’s tracheostomy, the nurse should instill saline in order toA) Decrease the client’s discomfortB) Reduce viscosity of secretionsC) Prevent client aspirationD) Remove a mucus plugThe correct answer is D: Remove a mucus plug112. A woman in her third trimester complains of severe heartburn. What is appropriateteaching by the nurse to help the woman alleviate these symptoms?A) Drink small amounts of liquids frequentlyB) Eat the evening meal just before retiringC) Take sodium bicarbonate after each mealD) Sleep with head propped on several pillowsThe correct answer is D: Sleep with head propped on several pillows113. A nurse is caring for a client who had a closed reduction of a fractured right wrist followed by the application of a fiberglass cast 12 hours ago. Which finding requires thenurse’s immediate attention?A) Capillary refill of fingers on right hand is 3 secondsB) Skin warm to touch and normally coloredC) Client reports prickling sensation in the right handD) Slight swelling of fingers of right handThe correct answer is C: Client reports prickling sensation in the right hand114. A newborn is having difficulty maintaining a temperature above 98 degrees Fahrenheit and has been placed in a warming isolate. Which action is a nursing priority?A) Protect the eyes of the neonate from the heat lampB) Monitor the neonate’s temperature
    C) Warm all medications and liquids before givingD) Avoid touching the neonate with cold handsThe correct answer is B: Monitor theneonate’s temperature115. The nurse is caring for a client with a myocardial infarction. Which finding requiresthe nurse’s immediate action?A) Periorbital edemaB) Dizziness spellsC) LethargyD) Shortness of breathThe correct answer is B: Dizziness spells116. A client is admitted with the diagnosis of pulmonary embolism. While taking a history, the client tells the nurse he was admitted for the same thing twice before, the lasttime just 3 months ago. Thenurse would anticipate the health care provider orderingA) Pulmonary embolectomyB) Vena caval interruptionC) Increasing the coumadin therapy to an INR of 3-4D) Thrombolytic therapyThe correct answer is B: Vena cava interruption117. A 70 year-old woman is evaluated in the emergency department for a wrist fractureof unknown causes. During the process of taking client history, which of these items should the nurse identify as related to the client’s greatest risk factors for osteoporosis?A) Menopause at age 50B) Has taken high doses of steroids for arthritis for many yearsC) Maintains an inactive lifestyle for the past 10 yearsD) Drinks 2 glasses of red wine each day for the past 30 yearsThe correct answer is B: Takes steroids for arthritis118. Decentralized scheduling is used on a nursing unit. A chief advantage of thismanagement strategy is that itA) Considers client and staff needsB) Conserves time for planningC) Frees the nurse manager to handle other prioritiesD) Allows for requests about special privileges
    The correct answer is A: Considers client and staff needs119. A newborn has hyperbilirubinemia and is undergoing phototherapy with a blanket.Which safety measure is most important during this process?A) Regulate the neonate’s temperature using a radiant heaterB) Withhold feedings while under the phototherapyC) Provide water feedings at least every 2 hoursD) Protect the eyes of neonate from the phototherapy lightsThe correct answer is C: Provide water feedings at least every 2 hours120. The nurse is at the community center speaking with retired people. To which comment by one of the retirees during a discussion about glaucoma would the nurse givea supportive comment to reinforce correct information?A) “I usually avoid driving at night since lights sometimes seem to make things blur.”B) “I take half of the usual dose for my sinuses to maintain my blood pressure.”C) “I have to sit at the side of the pool with the grandchildren since I can’t swim with thiseye problem.”D) “I take extra fiber and drink lots of water to avoid getting constipated.”The correct answer is D: “I take extra fiber and drink lots of water to avoid getting constipated.”121. On daily cleaning of a tracheostomy, the client coughs and displaces thetracheostomy tube. The nurse could have avoided this byA) placing an obturator at the client’s bedsideB) having another nurse assist with the procedureC) fastening clean tracheostomy ties before removing old tiesD) Withdraw catheter in a circular motionThe correct answer is C: fastening clean tracheostomy ties before removing old ties122. Which contraindication should the nurse assess for prior to giving a childimmunizations?A) Mild cold symptomsB) Chronic asthmaC) Depressed immune systemD) Allergy to eggsThe correct answer is C: Depressed immune system
  8. The nurse is teaching home care to the parents of a child with acute spasmodiccroup. The most important aspect of this care isA) Sedation as needed to prevent exhaustionB) Antibiotic therapy for 10 to 14 daysC) Humidified air and increased oral fluidsD) Antihistamines to decrease allergic responseThe correct answer is C: Humidified air and increased oral fluids124. A newborn delivered at home without a birth attendant is admitted to the hospital forobservation. The initial temperature is 35 degrees Celsius (95 degrees Fahrenheit) axillary. The nurse recognizes that cold stress may lead to what complication?A) Lowered BMRB) Reduced PaO2C) LethargyD) Metabolic alkalosisThe correct answer is B: Reduced PaO2125. In addition to standard precautions, a nurse should implement contact precautionsfor which client?A) 60 year-old with herpes simplexB) 6 year-old with mononucleosisC) 45 year-old with pneumoniaD) 3 year-old with scarlet feverThe correct answer is A: 60 year-old with herpes simplex126. Which of the following situations is most likely to produce sepsis in the neonate?A) Maternal diabetesB) Prolonged rupture of membranesC) Cesarean deliveryD) Precipitous vaginal birthThe correct answer is B: Prolonged rupture of membranes127. Which client is at highest risk for developing a pressure ulcer?A) 23 year-old in traction for fractured femurB) 72 year-old with peripheral vascular disease, who is unable to walk without assistanceC) 75 year-old with left sided paresthesia and is incontinent of urine and stoolD) 30 year-old who is comatose following a ruptured aneurysmThe correct answer is C: 75 year-old client with left sided paresthesia and is incontinent
    of urine and stool128. A new nurse manager is responsible for interviewing applicants for a staff nurseposition. Which interview strategy would be the best approach?A) Vary the interview style for each candidate to learn different techniquesB) Use simple questions requiring “yes” and “no” answers to gain definitive information*C) Obtain an interview guide from human resources for consistency inn interviewingeach candidateD) Ask personal information of each applicant to assure meeting of job demands The correct answer is C: Obtain an interview guide from human resources for consistency in interviewing each candidate129. A client who is 12 hour post-op becomes confused and says: “Giant sharks are swimming across the ceiling.” Which assessment is necessary to adequately identify thesource of this client’s behavior?A) Cardiac rhythm stripB) Pupillary responseC) Pulse oximetryD) Peripheral glucose stickThe correct answer is C: Pulse oximetry130. A client returns from surgery after an open reduction of a femur fracture. There is asmall bloodstain on the cast. Four hours later, the nurse observes that the stain has doubled in size. What is the best action for the nurse to take?A) Call the health care providerB) Access the site by cutting a window in the castC) Record the findings in the nurse’s notes onlyD) Outline the spot with a pencil and note the time and date on thecastThe correct answer is D: Outline the spot with a pencil and note the time and date on the cast131. A nurse assessing the newborn of a mother with diabetes understands thathypoglycemia is related to what pathophysiological process?A) Disruption of fetal glucose supplyB) Pancreatic insufficiencyC) Maternal insulin dependencyD) Reduced glycogen reservesThe correct answer is A: Disruption of fetal glucose supply
  9. The nurse is teaching a parent about side effects of routine immunizations. Which ofthe following must be reported immediately?A) IrritabilityB) Slight edema at siteC) Local tendernessD) Temperature of 102.5 FThe correct answer is D: Temperature of 102.5 F133. The parents of a toddler ask the nurse how long their child will have to sit in a carseat while in the automobile. What is the nurse’s best response to the parents?A) “Your child must use a care seat until he weighs at least 40 pounds.”B) The child must be 5 years of age to use a regular seat belt.C) “Your child must reach a height of 50 inches to sit in a seat belt.”D) “The child can use a regular seat belt when he can sit still.”The correct answer is A: “Your child must use a care seat until he weighs at least 40 pounds.”134. A 16 year-old boy is admitted for Ewing’s sarcoma of the tibia. In discussing his carewith the parents, the nurse understands that the initial treatment most often includesA) Amputation just above the tumorB) Surgical excision of the massC) Bone marrow graft in the affected legD) Radiation and chemotherapyThe correct answer is D: Radiation and chemotherapy135. A client complains of some discomfort after a below the knee amputation. Whichaction by the nurse is appropriate to do initially?A) Conduct guided imagery or distractionB) Ensure that the stump is elevated for the initial dayC) Wrap the stump snugly in an elastic bandageD) Administer opioid narcotics as orderedThe correct answer is B: Ensure that the stump is elevated for the initial day136. What is the best way that parents of pre-schoolers can begin teaching their childabout injury prevention?A) Set good examples themselvesB) Protect their child from outside influencesC) Make sure their child understands all the safety rulesD) Discuss the consequences of not wearing protective devices
    The correct answer is A: Set good examples themselves137. Which oxygen delivery system would the nurse apply that would provide the highestconcentrations of oxygen to the client?A) Venturi maskB) Partial rebreather maskC) Non-rebreather maskD) Simple face maskThe correct answer is C: Non-rebreather mask138. The nurse is teaching the mother of a 5 month-old about nutrition for her baby.Which statement by the mother indicates the need for further teaching?A) “I’m going to try feeding my baby some rice cereal.”B) “When he wakes at night for a bottle, I feed him.”C) “I dip his pacifier in honey so he’ll take it.”D) “I keep formula in the refrigerator for 24 hours.”The correct answer is C: “I dip his pacifier in honey so he”ll take it.”139. The nurse is performing an assessment on a client who is cachectic and has developed an enterocutaneous fistula following surgery to relieve a small bowel obstruction. The client’s total protein level is reported as 4.5. Which of the followingwould the nurse anticipate?A) Additional potassium will be given IVB) Blood for coagulation studies will be drawnC) Total parenteral nutrition (TPN) will be startedD) Serum lipase levels will be evaluatedThe correct answer is C: Total parenteral nutrition (TPN) will be started141. A newborn weighed 7 pounds 2 ounces at birth. The nurse assesses the newborn athome 2 days later and finds the weight to be 6 pounds 7 ounces. What should the nursetell the parents about thisweight loss?A) The newborn needs additional assessmentsB) The mother should breast feed more oftenC) A change to formula is indicatedD) The loss is within normal limitsThe correct answer is D: The loss is within normal limits
  10. During a situation of pain management, which statement is a priority to consider forthe ethical guidelines of the nurse?A) The client’s self-report is the most important considerationB) Cultural sensitivity is fundamental to pain managementC) Clients have the right to have their pain relievedD) Nurses should not prejudge a client’s pain using their own valuesThe correct answer is A: The client”s self report is the most important consideration143. A 35-year-old client of Puerto Rican-American descent is diagnosed with ovariancancer. The client states “I refuse both radiation and chemotherapy because they are ‘hot.’” The next action for the nurse to take is toA) Document the situation in the notesB) Report the situation to the health care providerC) Talk with the client’s family about the situationD) Ask the client to talk about the concerns about the “hot”treatmentsThe correct answer is D: Ask the client to talk about the concerns about the “hot” treatments144. Which of the following drugs should the nurse anticipate administering to aclient before they are to receive electroconvulsive therapy?A) BenzodiazepinesB) Chlorpromazine (Thorazine)*C) Succinylcholine (Anectine)D) Thiopental sodium (Pentothal Sodium)The correct answer is C: Succinylcholine (Anectine)145. A client is brought to the emergency room following a motor vehicle accident. Whenassessing the client one-half hour after admission, the nurse notes several physical changes. Which changes would require the nurse’s immediate attention?A) Increased restlessnessB) TachycardiaC) Tracheal deviationD) TachypneaThe correct answer is C: Tracheal deviation146. Which approach is a priority for the nurse who works with clients from many
    different cultures?A) Speak at least 2 other languages of clients in the neighborhoodB) Learn about the cultures of clients who are most often encounteredC) Have a list of persons for referral when interaction with these clients occurD) Recognize personal attitudes about cultural differences and real or expected biases The correct answer is D: Recognize personal attitudes about cultural differences and real or expected biases147. A client with chronic obstructive pulmonary disease (COPD) and a history ofcoronary artery disease is receiving Aminophylline, 25mg/hour. Which one of thefollowing findings by the nurse would require immediate intervention?A) Decreased blood pressure and respirations.B) Flushing and headache.C) Restlessness and palpitations.D) Increased heart rate and blood pressure.The correct answer is C: Restlessness and palpitations.148. The nurse is planning care for an 8 year-old child. Which of the following should beincluded in the plan of care?A) Encourage child to engage in activities in the playroomB) Promote independence in activities of daily livingC) Talk with the child and allow him to express his opinionsD) Provide frequent reassurance and cuddlingThe correct answer is A: Encourage child to engage in activities in the playroom.149. A pregnant client who is at 34 weeks gestation is diagnosed with a pulmonary embolism (PE). Which of these mediations would the nurse anticipate the health careprovider ordering?A) Oral Coumadin therapyB) Heparin 5000 units subcutaneously b.i.d.C) Heparin infusion to maintain the PTT at 1.5-2.5 times the control valueD) Heparin by subcutaneous injection to maintain the PTT at 1.5 times the control valueThe correct answer is D: Heparin by subcutaneous injection to maintain the PTT at 1.5 times the control value150. The nurse is caring for a client with Hodgkin’s disease who will be receiving radiation therapy. The nurse recognizes that, as a result of the radiation therapy, the clientis most likely to experience
    A) High feverB) NauseaC) Face and neck edemaD) Night sweatsThe correct answer is B: Nausea151. While assessing the vital signs in children, the nurse should know that the apical heart rate is preferred until the radial pulse can be accurately assessed at about what age?A) 1 year of ageB) 2 years of ageC) 3 years of ageD) 4 years of ageThe correct answer is B: 2 years of age152. Which of these clients, who all have the findings of a board-like abdomen, wouldthe nurse suggest that the health care provider examine first?A) An elderly client who stated that “My awful pain in my right side suddenly stoppedabout 3 hours ago.”B) A pregnant woman of 8 weeks newly diagnosed with an ectopic pregnancyC) A middle-aged client admitted with diverticulitis and has taken only clear liquids forthe past weekD) A teenager with a history of falling off a bicycle and did not hit the handle bars The correct answer is A: An elderly client who stated that “My awful pain in my right side suddenly stopped about 3 hours ago.”153. A client with a panic disorder has a new prescription for Xanax (Alprazolam). Inteaching the client about the drug’s actions and side effects, which of the following should the nurse emphasize?A) Short-term relief can be expectedB) The medication acts as a stimulantC) Dosage will be increased as toleratedD) Initial side effects often continueThe correct answer is A: Short-term relief can be expected154. Which of these questions is priority when assessing a client with hypertension?A) “What over-the-counter medications do you take?”B) “Describe your usual exercise and activity patterns.”C) “Tell me about your usual diet.”D) “Describe your family’s cardiovascular history.”
    The correct answer is A: “What over-the-counter medications do you take?”155. During a routine check-up, an insulin-dependent diabetic has his glycosylated hemoglobin checked. The results indicate a level of 11%. Based on this result, whatteaching should the nurse emphasize?A) Rotation of injection sitesB) Insulin mixing and preparationC) Daily blood sugar monitoringD) Regular high protein dietThe correct answer is C: Daily blood sugar monitoring156. Which of these clients would the nurse monitor for the complication of C. difficilediarrhea?A) An adolescent taking medications for acneB) An elderly client living in a retirement center taking prednisoneC) A young adult at home taking a prescribed amino glycosideD) A hospitalized middle aged client receiving clindamycinThe correct answer is D: A hospitalized middle aged client receiving clindamycin157. The nurse is assessing a comatose client receiving gastric tube feedings. Which ofthe following assessments requires an immediate response from the nurse?A) Decreased breath sounds in right lower lobeB) Aspiration of a residual of 100cc of formulaC) Decrease in bowel soundsD) Urine output of 250 cc in past 8 hoursThe correct answer is A: Decreased breath sounds in right lower lobe158. The nurse is preparing to take a toddler’s blood pressure for the first time. Which ofthe following actions should the nurse do first?A) Explain that the procedure will help him to get wellB) Show a cartoon character with a blood pressure cuffC) Explain that the blood pressure checks the heart pumpD) Permit handling the equipment before putting the cuff in placeThe correct answer is D: Permit handling the equipmentbefore putting the cuff in place159. A 72 year-old client is scheduled to have a cardioversion. A nurse reviews the client’s medication administration record. The nurse should notify the health care
    provider if the client received which medication during the preceding 24 hours?A) digoxin (Lanoxin)B) diltiazam (Cardizem)C) nitroglycerine ointmentD) metoprolol (Toprol XL)The correct answer is A: digoxin (Lanoxin)160. To prevent drug resistance common to tubercle bacilli, the nurse is aware that whichof the following agents are usually added to drug therapy?A) Anti-inflammatory agentB) High doses of B complex vitaminsC) Amino glycoside antibioticD) Two anti-tuberculosis drugsThe correct answer is D: Two anti-tuberculosis drugs

Leave a Comment

Scroll to Top